Você está na página 1de 36

GAZETA DE FISICA

R E V I S T A D O S E S T U D A NT E S D E F Í S I C A E
DOS FÍSICOS E TÉCNICO-FÍSICOS PORTUGUESES

VOL. I, FASC. 1
OUTUBRO, 1946

RADIOGRAFIA OBTIDA NO
LABORATÓRIO DE FÍSICA
DA F. C. L.
POR UM DOS ALUNOS
DO CURSO DE FÍSICA
F. Q. N.
GAZETA DE FÍSICA
Vol. I, Fasc. 1 Outubro de 1946
RESPONSÁVEIS DAS SECÇÕES
SUMÁRIO
1. TRIBUNA DE FÍSICA

1. Tribuna da Física Armando Gibert


Em nome da Direcção por Armando Gibert 1 2. O ENSINO MÉDIO DA FÍSICA
Ensino e investigação por Armando Cyrillo Soares 3
J. Xavier de Brito
2. Ensino Médio da Física
3. 0 ENSINO SUPERIOR DA FÍSICA
Exames de Aptidão por Rómulo de Carvalho .
F. Soares David e Lídia Sal-
3. Ensino Superior da Física gueiro.
Ensino prático da Física F. Q. N. por Lídia Salgueiro 6 4. EXAMES DO ENSINO MÉDIO

4. Exames do Ensino Médio Rómulo de Carvalho


Pontos de Exames de Aptidão. Resol. de Rómulo de Carvalho 9 5. EXAMES UNIVERSITÁRIOS
5. Exames Universitários Carlos Braga, João de Almeida
Pontos de Exame. Resoluções de Glaphyra Vieira 10 Santos, Jose' Sarmento e Gla-
phyra Vieira
7. Problemas propostos
Palavras prévias por Amaro Monteiro . 12 6. PROBLEMAS DA INVESTIGAÇÃO EM
FÍSICA
8. Divulgação e Vulgarização Manuel Valadares
Como se mediu a carga do electrão por Rómulo de Carvalho 13
7. PROBLEMAS PROPOSTOS
9. História e Antologia Amaro Monteiro
Engenharia atómica? por Theodore Von Kármán 18
8. DIVULGAÇÃO E VULGARIZAÇÃO
Gabriel Lippmann por Maria Helena Blanc de Sousa 20
Rómulo de Carvalho
10. Química
9. HISTÓRIA E ANTOLOGIA
Origem e objectivo desta secção por Marieta da Silveira 20
Nomenclatura química por Alice Maia Magalhães 21 Lídia Salgueiro
Pontos de Exames de Aptidão 22 10. QUÍMICA
Problemas de Exames Universitários 23 Alice Maia Magalhães, Afonso
11. A Física nas suas aplicações Morgenstern e Manieta da Sil-
veira.
A Física e a Engenharia Civil por Manuel Rocha 24
11. A FÍSICA NAS SUAS APLICAÇÕES
12. Informações Várias 31
Armando Gibert

A matéria de cada artigo é tratada sob a inteira responsabilidade do autor 12. INFORMAÇÕES VÁRIAS

Direcção

Correspondência dirigida a DIRECÇÃO


GAZETA DE FÍSICA Jaime Xavier de Brito
Laboratório de Física, F. C. L. Rómulo de Carvalho
Armando Gibert
R. da Escola Politécnica—LISBOA
Lídia Salgueiro
NÚMERO AVULSO ESC. 10$00 SECRETÁRIO
Condições de assinatura: Maria Augusta Pérez Fernández
4 Números (1 ano) Esc. 30$00 PROPRIEDADE E EDIÇÃO:
12 Números (3 anos) Esc. 75$00 Gazeta de Matemática, Lda.

Dep.: LIVRARIA ESCOLAR EDITORA — R. da Escola Politécnica, 68-72 -Tel. 6 4040 — LISBOA

Consulte a lista de preços dos nossos anúncios

Tipografia de Matemática, Lda.-Lisboa.


GAZETA DE FÍSICA
Fundador: ARMANDO GIBERT

Direcção: J. Xavier de Brito — Rómulo de Carvalho — Armando Gibert— Lídia Salgueiro

Vol. I, Fasc. 1 O u t u b r o de 1 9 4 6

1. TRIBUNA DA FÍSICA
EM NOME DA DIRECÇÃO

ARMANDO GIBERT

A «Gazeta de Física» tem por primeiro e encontrarão associando físicos às suas emprê-
grande objectivo contribuir activamente para sas (como fazem correntemente lá fóra). Lu-
o desenvolvimento e elevação dos estudos de taremos ainda por condições de vida dignas,
Física em Portugal em todos os graus de en- únicas capazes de assegurar um desempenho
sino, assim como para o esclarecimento dum eficiente e elevado de qualquer profissão, em
público mais vasto sôbre a posição real da particular da nossa, tão prejudicialmente ina-
intervenção da Física na vida moderna e sô- proveitada entre nós.
bre a acção do nível científico dos físicos e Dentro dêstes moldes não deixaremos de
técnico-físicos no ritmo e na independência imprimir à nossa revista uma orientação bem
do progresso industrial do nosso País. definida, mas agradecemos a todos os que con-
Estas palavras iniciaram a nossa primeira cordem com aquêles fins que enriqueçam os
circular. Mais adiante, propúnhamos, na mes- nossos projectos com sugestões, que serão
ma, a luta pela criação dum Corpo Nacional sempre bem recebidas, e que contribuam para
de Físicos e Técnicos Científicos. tornar o nosso serviço informativo tão com-
É isto, sem dúvida, o resumo dum programa pleto quanto possível. Desejamos também
e julgamos oportuno procurar desde já com- pôr as páginas da Gazeta de Física ao serviço
pletá-lo e desenvolvê-lo um pouco mais... do maior número de físicos ou amigos da
Para facilitar muitos dos nossos outros Física e para defesa das idéias mais diversas,
objectivos, procuraremos realizar por meio da mas reservamo-nos o direito de excluir todas
Gazeta de Física as condições, tão desejáveis as contribuições que, de qualquer modo, pelo
no nosso meio, de coordenação de iniciativas seu espírito ou pela sua forma, nos pareçam
e esforços dispersos e de colaboração entre susceptíveis de contrariar os nossos objectivos
os professores dos vários graus de ensino e fundamentais. Por vezes o nosso ponto de
os seus antigos alunos. Tentaremos também vista será errado, e talvez aquele que rejeitar-
promover o maior interesse de todos pela mos seja porventura o bom. No entanto es-
«profissão» de físico, em particular dos nos- peramos que tais êrros se dêem raramente,
sos industriais, pelas vantagens que muitos mas a atitude que nos leva a correr um tal
1
Vol. I, Fasc. 1 GAZETA DE FÍSICA Outubro, 1946

risco é a única que nos parece compatível 3.°) Estamos nós seguros de que reunimos
com a eficácia da nossa acção e, por isso, pouco mais do que as condições mínimas para
havemos de a manter com intransigência. um desempenho activo e progressivo da nossa
Oxalá a franqueza desta explicação evite que profissão?
algum dos nossos amigos se sinta melindrado Se as respostas ao que precede forem ne-
no dia em que, por fôrça da doutrina exposta, gativas, podemos admitir que assim seja?
sejamos levados a não publicar um dos seus não será nossa obrigação aprofundar as causas
artigos ou solicitemos a supressão ou a modi- de tais negativas? como conseguir que os
ficação de alguma das suas partes. actuais e futuros alunos de Física possam um
Havemos de dirigir-nos, muitas vezes, a dia responder-lhe diferentemente?
pessoas cujas relações com a Física são des- Crêmos ter estabelecido assim um imenso
conhecidas ou negadas entre nós, mas também programa de luta. Oxalá êle corresponda,
não descuraremos a tarefa fundamental de como esperamos, às aspirações mais elevadas
criar uma consciência colectiva dos nossos do maior número possível dos mais influentes
físicos (ou daqueles que assim deveriam con- dos nossos colegas. Contamos com a colabo-
siderar-se). Para isso é essencial encontrar- ração activa de todos, desejamos conhecer os
mos a coragem de pôr a nú, sem desfalecer, mais variados pontos de vista e aguardamos
as verdades dolorosas que se opõem ao reco- sugestões construtivas dos que sintam, como
nhecimento da importância da nossa profissão. nós, a urgência premente de realizar aquêle
O caminho será longo, e só poderemos atingir programa.
a meta se procedermos constantemente a um Mas terá êle ficado claramente posto? In-
profundo exame de consciência profissional e sistimos:
se fizermos e aceitarmos todas as críticas com 1.°) A Licenciatura em Ciências Físico-
uma objectividade totalmente impessoal -Químicas não é um curso de Física. Quere-
Alguns perguntarão: mas que devemos nós mos uma «Licenciatura em Física», indepen-
fazer? não tirámos nós um curso com uma dente, única capaz de dar aos nossos futuros
colegas um nível profissional elevado, com-
boa classificação? não procuramos desempe-
parável ao dos físicos estrangeiros.
nhar-nos o melhor possível, e com satisfação
2.°) Os princípios gerais que regulam a
dos nossos superiores, da tarefa profissional
transmissão da Ciência e a verificação do
que nos cabe?...
aproveitamento, nas nossas Faculdades, são
Desejamos acentuar, já neste primeiro nú-
falsos, regressivos, ineficientes e improdutivos.
mero, que queremos viver com os olhos pos-
A Licenciatura em Física (como aliás todas
tos no futuro, que as nossas críticas ao pas-
as outras) precisa de ser organizada em bases
sado serão sêcamente realistas (tendo apenas
novas, aumentando o ensino prático, redu-
em vista evitar repetição de êrros e de modo
zindo ao mínimo as aulas magistrais, criando
algum culpar aqueles que os cometeram) e
Seminários e Colóquios, não tirando à Física
que não admitiremos chicanas pessoais nas
que se ensina a Vida que lhe é própria e que
nossas páginas, pois nem as pessoas nem as
mais naturalmente pode despertar o entu-
suas susceptibilidades nos interessam.
siasmo criador dos alunos, mas não descurando
Por isso, em tôrno da pergunta acima for-
tão pouco o papel essencial da Hipótese nem
mulada e das objecções interrogativas que se
o valor estimulante da Teoria.
lhe seguem, sugerimos os seguintes motivos
de meditação: 3.°) Se não tirámos um curso de Física,
1.°) Tirámos nós, de facto, um curso de Física? se a Licenciatura em Físico-Químicas tem
2.°) Existirá, efectivamente, uma relação apenas 4 cadeiras de Física, uma das quais
de causa a efeito entre aquilo que ficámos semestral, se as condições dêsse ensino redu-
sabendo para tôda a vida e as notas que os zido estão longe de lhe dar uma eficácia má-
nossos exames mereceram? xima, se a prática experimental é uma com-
2
Vol. I, F a s c . 1 GAZETA DE FÍSICA Outubro, 1946

pleta ficção, como poderemos nós ter uma da importância que nós sabemos que ela tem.
cultura e capacidade profissional do nível da Êste trabalho não deve sofrer interrupções e
dos nossos colegas estrangeiros e, portanto, cada um deverá realizá-lo de acôrdo com as
como poderá ela ser suficiente para o desem- tendências pessoais e as suas possibilidades,
penho activo e progressivo da nossa profissão? mas seria extraordináriamente vantajoso que
Não o é de facto, mas, quanto a nós, pensa- os esforços individuais se apoiassem em tare-
mos que só nos deve honrar reconhecê-lo e fas colectivas, por exemplo, em centros de
proclamá-lo. Mas isto não basta... de estudo ligados de preferência às Faculda-
Uma vez definida a situação de inferioridade des, escrevendo na Gazeta de Física sôbre
em que fomos colocados por virtude de cau- temas de ensino, quer teóricos, quer práticos,
sas estranhas à nossa vontade, é o nosso pri- colaborando em ciclos de conferências, etc.
meiro dever sacudir as algemas da comodidade Ventilámos assim algumas idéias que, pro-
e da suficiência e lançar mãos à obra de com- vàvelmente, constituíam já motivo de preocu-
pletar e elevar a nossa cultura profissional, pação para muitos de nós. Esperamos que
lutando ao mesmo tempo pela criação de con- à «Gazeta de Física» caiba o importante papel
dições favoráveis para as gerações futuras. de ser o agente de ligação, entre cada um de
É êste o nosso primeiro dever, pois só as- nós e todos os outros, na preparação e no es-
sim nos poderemos impor como classe profis- tudo da acção coordenada que devemos desen-
sional e exigir dos outros o reconhecimento volver sem tardar.

ARMANDO GIBERT

ENSINO E INVESTIGAÇÃO

ARMANDO CYRILLO SOARES

No século que decorre, não passa de uma actividade científica como condição indispen-
banalidade a afirmação de que só devem con- sável para poderem ser incluidos na lista das
siderar-se verdadeiramente civilizadas as na- nações civilizadas e, conseqüentemente pode-
ções que ao desenvolvimento da Ciência dedi- rem, de direito e não parasitàriamente, usu-
cam boa parte de suas energias e de seus ren- fruir todos os benefícios de riqueza, de pode-
dimentos. rio e de elevação da Humanidade, em quinhões
Na verdade, são essas nações que, àlém de tanto mais largos quanto mais valiosas forem
criarem, para a Humanidade, riquezas e pos- as respectivas contribuïções para a construção
sibilidades cada vez maiores, simultâneamente científica, isto é, para o progresso da Civili-
tornam o Homem cada vez mais digno de go- zação.
zar os respectivos benefícios. Em Portugal, como no resto do mundo, de
Os esforços feitos pelos homens de ciência há muito se estabeleceram as idéias que aí
para descobrirem os tesoiros que a Natureza ficam, se reconheceu o sentido em que deve-
contém, procederem ao inventário e avaliação mos marchar, como nação desejosa de digni-
dêsses tesoiros e garantirem o domínio hu- ficar-se, e se apreendeu a responsabilidade
mano dos mesmos, visam a contínua amplia- que colectivamente assumimos, se não nos
ção da Ciência, como sólido alicerce para o aplicarmos esforçadamente a partilhar na ta-
desenvolvimento da Civilização. refa imposta à Humanidade para seu engran-
Estas idéias reconhecidas universalmente decimento próprio pelo engrandecimento da
como assêrtos, a tôdos os povos indicam a Ciência.
3
Vol. I, Fasc. 1 GAZETA DE FÍSICA Outubro, 1946

Assim, há muito tempo que se reconhece e actividades científicas, quer estas visem as apli-
se lamenta o atrazo em que nos deixamos cações proveitosas imediatas, quer tomem o ca-
cair relativamente a outras nações e se repe- rácter de investigação tendente a aumentar o sa-
tem tentativas de projectos para vencer o re- ber, ainda que sem aplicação utilitária imediata.
ferido atrazo; não faltam aos nossos estudio- Pouco tempo depois (1915) até os laborató-
sos, de um modo geral, as qualidades neces- rios liceais adquiriam animado funcionamento
sárias aos trabalhadores científicos que, em com a introdução no ensino secundário dos
ambiente próprio, se revelam em medida tão trabalhos individuais educativos para os alunos
boa, ou até melhor, do que aquela em que os dos cursos complementares.
cientistas de outros países as possuem; mas O ensino científico melhorava, tanto no
o aludido atrazo persiste, principalmente tal- grau secundário como no superior; deixava de
vez, porque neste assunto, como em muitos reduzir-se com demasiada freqüência à
outros, nós portugueses nos mostramos pouco exposição oral ou escrita do professor ou do
dados a persistir na execução de um plano, compêndio, nem sempre fàcilmente apreensí-
destinado a resolver certo problema, sem o veis por falta de referências ou estabelecida
submetermos a freqüentes e amplas modifica- relação com factos conhecidos pelo aluno
ções, quer no critério orientador da execução, como realidades por êle observadas; com
quer até no enunciado do problema, ou seja, efeito, o trabalho laboratorial, consistindo no
na definição do objectivo a atingir; e em tais contacto com realidades — porventura, algu-
condições o rendimento das energias aplicadas mas apenas — dá-nos, contudo, a capacidade
é, em geral, mesquinho e desanimador. de apreender claramente por simples descrição
É sabido que o desenvolvimento da Ciência outros factos reais como se os observássemos.
se alcança pela difusão do Saber e pelo seu Infelizmente os benefícios que a legislação
acrescentamento, isto é, pelo ensino e pela de 1911 e 1915 trouxe ao ensino científico,
investigação. Que se passa em Portugal nes- em grande parte se perderam já, quer por
tes campos de actividade nacional? modificações da referida legislação, quer por
A curto prazo sôbre a proclamação da Re- deficiências de instalação, de pessoal e de equi-
pública, promulgou-se a reforma universitária pamento que não têem aumentado na propor-
de 1911 que trouxe ao ensino superior notá- ção em que tem crescido a população das
vel melhoria, permitindo uma maior eleva- escolas. A necessidade de reforma dos ensinos
ção dos programas, dando ao ensino prático liceal e superior é proclamada por todos que
uma largueza e importância que nunca antes pelo assunto se interessam. Quando e em
tivera tão nìtidamente marcada e impondo que sentido se efectivarão essas reformas?
aos alunos, a par da concessão de grande li- Oxalá que em breve e benèficamente se
berdade na regulação das suas tarefas anuais, reforme, tendo presente que uma fértil acti-
tanto no respeitante à escôlha dos cursos a vidade de investigação científica, actividade
seguir como no referente às provas de exame característica de nação civilizada, exige como
a prestar, o sentimento da própria responsa- base um bom ensino científico.
bilidade na marcha e eficiência dos seus tra- Em 1929, coroando esforços que de mais
balhos escolares e na apreciação dos resulta- longe se vinham efectuando, foi criada a ins-
dos obtidos. tituïçào que hoje se designa «Instituto para a
Os laboratórios dos cursos de ciências expe- Alta Cultura», tendo como um dos seus
rimentais perderam o aspecto de museus ou objectivos o desenvolvimento da investigação
de depósitos de material, que muitos apresen- científica no nosso país. Por esta instituïção têm
tavam na maior parte da sua extensão, e sido concedidas bolsas de estudo no estran-
passaram a ser locais de trabalho dos alunos, geiro e no país e têm sido subsidiados alguns
iniciando-se êstes nas técnicas experimentais laboratórios e outros institutos científicos para
e desenvolvendo-se em muitos o gôsto pelas aquisição de material, publicações, etc.
4
Vol. I, Fasc. 1 GAZETA DE FÍSICA Outubro, 1946

De um modo geral os bolseiros enviados ao investigação experimental, aplicando técnicas


estrangeiro têm cumprido a sua missão com das mais delicadas e modernas com êxito muito
resultado conveniente; alguns, em proporção apreciável.
muito notável, levaram a cabo os seus traba- O aproveitamento da competência adquirida
lhos em meios científicos de alto nível por por êstes cientistas é o fim que se teve em
forma verdadeiramente brilhante. O êxito dos vista, quando se promoveu com apreciável
trabalhos dêsses bolseiros, os títulos científicos dispêndio de dinheiros a sua formação. Mas
alcançados por muitos, a consideração e a êsse aproveitamento que é um direito do
estima que a alguns têm sido manifestadas Estado, impõe a êste o dever de garantir àqueles
por cientistas da mais elevada categoria nos cientistas as condições de trabalho científico
meios científicos em que fizeram os seus está- a que êles se lançarão com entusiasmo, e as
gios, constituem a honrosa demonstração de condições económicas de vida sem cuja segu-
que não será por falta de gente capaz para a rança não pode ter apreciável rendimento a
obra de desenvolvimento da Ciência que actividade científica.
Portugal se manterá no degradante atrazo Terá o «Instituto para a Alta Cultura»
que, em relação às nações civilizadas, se tem possibilidade de manter em actividade científica
reconhecido e lamentado. dedicada e em condições de razoável segurança
Alguns dos mais distintos bolseiros enviados económica os investigadores cuja formação
ao estrangeiro, após o seu regresso a Portugal subsidiou? O contrário levaria a considerar
têm-se dedicado à meritória obra de instruir o seu funcionamento sujeito a ter como resul-
outros estudiosos nas técnicas modernas de tado principal desperdícios de capitais e de
investigação em que lá fora se adestraram, valores que poderia tornar precária a utilidade
dando assim bom rendimento ao capital que para da instituïção que tantas esperanças de pro-
a sua preparação o Estado tenha dispendido. gresso real nos havia trazido. Esperemos que
Foi por êste processo que alguns bolseiros no tal não se verifique.
País conseguiram já produzir trabalhos de
A. CYRILLO SOARES
PROF. CATEDRÁTICO DA F. C. L.

2. ENSINO MÉDIO DA FÍSICA


EXAMES DE APTIDÃO

RÓMULO DE CARVALHO

Pensou a Direcção da «Gazeta de Física» Tem-se tratado, incidentalmente em artigos


que deveria interessar a grande parte dos de imprensa e freqüentemente em conversas
seus leitores a publicação dos pontos escritos nas escolas, desta situação de «contrasenso»,
de Física que saem nos exames de aptidão às de os alunos terminarem o exame do 7.° ano
Escolas Superiores do País. É nosso desejo do liceu e serem submetidos, dias depois, a
que, tudo quanto respeite à Física, tenha ca- novo exame que versa as mesmas matérias.
bimento nestas páginas. Os professores liceais sentem a honra ferida
Em particular, o assunto dos pontos de enquanto os professores universitários ficam
aptidão é de interêsse que supera o de muitas boquiabertos com as ignorâncias que alguns
questões de ensino porque ilumina vivamente alunos patenteiam nos pontos. Em nosso
o lado defeituoso de algumas delas. entender não se trata duma questão de
5
Vol. I, Fasc. 1 GAZETA DE FÍSICA Outubro, 1946

honra ferida mas sim dum problema de en- para se poder ingressar numa Faculdade.
sino. Basta o 6.° ano pois a maioria das perguntas
Em qualquer assunto que se discuta todos feitas referem-se a modestíssimos conhecimen-
têm a sua verdade e todos a tomam por se- tos do curso geral. Quanto aos problemas,
gura. O que é, porém, seguro, é o facto. êles são da mesma categoria das perguntas:
E o facto é êste: as Escolas Superiores diri- propositadamente simples. Quem conheça o
gem, aos alunos que nelas desejam ingressar, assunto que os leia e os aprecie desinteres-
perguntas tão irrisórias que nenhum professor sadamente.
do liceu se disporia a pô-las num exame do E depois? Que resulta de tudo isto? Um
7.° ano. Por exemplo estas: Como se de- número de reprovações tão impressionante
monstra o princípio de Arquimedes? Que é que chega, só por si, para justificar a conti-
potência duma lente? Que é um calorímetro? nuação do «contrasenso» dos ditos exames.
Que é liquefacção? Chega-se a pensar que o Não está aqui, patente aos olhos de todos,
7.° ano do liceu é muito pouco necessário um motivo de meditação?
RÓMULO DE CARVALHO
PROF. DO LICEU CAMÕES

3. ENSINO SUPERIOR DA FÍSICA


ENSINO PRÁTICO DA FÍSICA F.Q.N.

LÍDIA SALGUEIRO

Na organização do programa de trabalhos O quadro da página seguinte facilita a com-


práticos do curso de Física F. Q. N., prepa- paração das matérias ensinadas nas referidas
ratório para ciências médicas e biológicas, na escolas.
Faculdade de Ciências de Lisboa, tem-se pro- Limitamo-nos a indicar os principais assun-
curado atender, na medida do possível, aos tos tratados não se especificando em geral
interêsses dos alunos para a sua vida futura. dentro de cada capítulo todos os trabalhos
O projecto traçado inicialmente tem sofrido realizados, para não tornar muito longa esta
algumas modificações para as quais contribuí- descrição.
ram principalmente dois factores: conheci- No entanto, o leitor que deseje obter uma
mento de cursos práticos análogos ministrados informação mais completa poderá consultar os
noutras Universidades e inquéritos feitos aos livros a que nos referimos anteriormente e que
alunos no fim de cada ano. são respectivamente:
Consultando livros de Física para estudantes Physics for medical students —J. S. Rogers.
que se destinam a médicos ou a biólogos Travaux pratiques de Physique —C. Simon.
verificámos que os programas seguidos em Einführung in das Physikalische Praktikum
várias Universidades são em geral seme- —Christian Gerthen und Max Pollermann.
lhantes. Há um facto que nos impressiona quando
Para concretizar vamos começar por indicar consultamos êstes livros: a abundância de
resumidamente quais os capítulos da Física a trabalhos realizados durante cada ano e por-
que se dedica especial atenção nas Universi- tanto a importância que naquêles países se dá
dades de Melbourne (Austrália), de Dijon ao ensino prático. Infelizmente no nosso país
(França) e no Physikalische Institut (Berlim). êste ensino é muito mais deficiente em virtude
6
Vol. I, Fasc. 1 GAZETA DE FÍSICA Outubro, 1946

Trabalhos Universidades

Mecânica Medida de comprimentos Dijon Phys. Institut


Medida de tempos Dijon Phys. Institut
Pesagens e densidades Dijon Phys. Institut
Tensão superficial Dijon Phys. Institut
Medida de fôrças Dijon Phys. Institut
Oscilações Dijon
Pressões Dijon
Viscosidade Dijon
Solubilidade Dijon
Osmose Melbourne
Pressão do sangue Melbourne
Estado coloidal Dijon Phys. Institut Melbourne

Calor Termometria Dijon


Calorimetria Dijon
Higrometria Dijon
Propagação do calor Dijon

Óptica Espelhos Dijon


Lentes Dijon Phys. Institut
Refractómetros Dijon Phys. Institut
Aberrações Dijon
Microscópio Dijon Phys. Institut Melbourne
Polarimetria Dijon Phys. Institut
Fotometria subjectiva Dijon Phys. Institut
Fotometria com fotopar Dijon Phys. Institut
Rêdes de difracção Dijon Phys. Institut
Interferência Dijon
Espectroscopia do visivel Dijon Phys. Institut
Colorimetria Dijon
Dupla refracção Dijon
Ultravioleta Melbourne
Visão humana Melbourne
Coeficiente absorção com fotopar Phys. Institut

Electricidade Medida de resistências Dijon Phys. Institut


Electrólise Dijon Phys. Institut
Galvanómetro balístico Dijon
Electrómetros Dijon
Fotoelectricidade Dijon
Estudo de um díodo Dijon
Estudo de um tríodo Dijon
Correntes de alta freqüência Melbourne
Raios X Phys. Institut Melbourne
Radioactividade Melbourne

do grande número de alunos de cada turma e desfavoráveis conseguiu-se realizar no período


do reduzido número de aulas práticas semanais. de 1941-1946 a série de trabalhos que a seguir
Acresce ainda que muitas vezes somos impe- indicamos:
didos de realizar certas experiências devido à Óptica:
insuficiência de meios de que dispomos. Espectroscopia do visível: (observação de
A-pesar dêste conjunto de circunstâncias espectros contínuos e de riscas; espectros de
7
Vol. I, Fasc. 1 GAZETA DE FÍSICA Outubro, 1946

absorção e sua importância na biologia e na de modificar a apresentação de certos assuntos


medicina). de modo a produzirem maior interêsse nos
Espectrografia do ultravioleta (obtenção do alunos.
espectro do vapor de mercúrio, papel dos fil- Como se vê todos os trabalhos realizados
tros, designadamente dos vidros dos óculos no curso de Física F. Q. N. da Faculdade de
usados pelos médicos para protecção do Ciências de Lisboa são também executados pelo
ultravioleta). menos numa das três Universidades citadas.
Fluorescência produzida pelo ultravioleta; Muitas das experiências mencionadas englo-
luz de Wood. bam outras que não é possível executar
Medida do coeficiente de absorção do vidro isoladamente; assim, por ex., no estudo da
com um fotopar. electricidade os alunos adquirem algumas
Microscópio: observação de preparações, noções práticas sôbre transformadores e ainda
amplificação do microscópio, determinação da sôbre medidas de correntes e tensões contí-
distância de dois pontos numa preparação e nuas e alternadas; designadamente no estudo
desenhos com uma câmara clara; microfo- da lâmpada de dois electródios há oportuni-
tografia. dade de ampliar as escalas de voltímetros e
Calorimetria: amperímetros, o que constitui portanto uma
Determinação da temperatura com um ter- aplicação prática dêstes problemas; na elec-
mopar. trólise há necessidade de se realizarem opera-
Electricidade: ções de pesagem, etc.
Lâmpada de dois electródios (determinação Consegue-se assim atenuar em parte as
da corrente de saturação; influência da varia- deficiências que nos são impostas pela limita-
ção da corrente de aquecimento. Importância ção do tempo de aulas.
dêste trabalho para a compreensão do funcio- Acrescentemos ainda que algumas das
namento dos rectificadores e de uma ampola experiências que ficam por realizar fazem
de raios X). parte do programa de Química F. Q. N.
Lâmpada de três electródios (papel da grelha seguido nesta Faculdade (polarimetria, refrac-
e funcionamento como amplificador). tómetria, colorimetria e viscosidade), o que
Electrólise: determinação da carga eléctrica torna portanto inútil repetí-las no curso de
do electrão. Física.
Raios X: explicação da montagem duma Embora o nosso programa de ensino prático
instalação; radioscopia de vários objectos; esteja muito longe de ser completo,
influência da tensão e da corrente; variação permite no entanto, aliado a experiências
da absorção com o número atómico; radio- realizadas nas aulas teóricas (polarização,
grafia; ionização produzida com um feixe de dupla refracção, difracção, etc.), dar aos
raios X (detecção com um electroscópio); alunos uma visão geral da importância e da
dosimetria com um par fotoeléctrico. utilidade da Física para a vida a que se
Radioactividade: experiências com um elec- destinam.
troscópio de Wulf (ionização produzida por Seria no entanto para desejar que se execu-
partículas alfa e determinação da corrente de tassem muito mais experiências e que os
saturação); observação de cintilações produ- alunos tomassem uma parte mais activa nos
zidas por partículas alfa com o espintariscópio trabalhos práticos, o que só poderá conse-
de Crookes. guir-se com o aumento do número de aulas e
Os inquéritos a que nos referimos no de assistentes de Física, e com a aquisição de
comêço, têm actuado principalmente no sentido maior quantidade de material.

LÍDIA SALGUEIRO
ASSIST. DE FÍSICA DA F. C. L.

8
Vol. I, Fasc. 1 GAZETA DE FÍSICA Outubro, 1946

4. EXAMES DO ENSINO MÉDIO


PONTOS DE EXAMES DE APTIDÃO

Faculdade de Ciências 9 — Enuncie a lei de Lenz e indique quais as


suas conseqüências.
1 — O valor da velocidade de uma massa indefor- 10 — Escreva o que sabe sôbre os raios X.
mável de 20,0 quilogramas em movimento de trans-
lacção passou, em dado intervalo de tempo, de 30,0 m/s Escola Superior de Farmácia
para 50,0 m/s. Calcule o trabalho de tôdas as fôrças 11 — De que factores depende a evaporação de um
que actuaram a aludida massa no intervalo de tempo líquido? Escreva a fórmula que traduz essa depen-
considerado e exprima o resultado do seu cálculo em dência.
joules e também em quilogrâmetros. (R: 16000 joules).
12 — Qual a marcha dos raios luminosos nos pris-
2 — Um utensílio doméstico, alimentado pela cor- mas triangulares de secção recta e de catetos iguais?
rente eléctrica, suporta a seguinte inscrição: «250 W»
Que significa essa inscrição? 13 — Que entende por calorímetro? Para que
Calcule a potência consumida pelo referido utensí- serve? Escreva a fórmula que se aplica em trabalhos
lio, exprimindo-a em unidades do sistema métrico. com êste aparelho.
(R: 25,51 Kgm/s). 14 — Quais são as condições de máxima sensibili-
3 — Sabendo-se que, na queda livre dum grave, dade para as balanças?
cresce continuamente a energia cinética do mesmo, 15 — Que entende por agulhas astáticas? Para
como explica que a referida queda concorde com o que servem?
princípio da conservação da energia?
16 — Escreva a fórmula que traduz a lei de Joule,
4 — Indique pela ordem crescente das respectivas em electricidade, indicando o significado das letras.
freqüências os diversos tipos de radiações electroma-
gnéticas que conhece, e aponte, para cada tipo, alguma 17 — Um gás tem a 10 graus centígrados a pressão
das suas propriedades características ou das suas de 720 mm. Que pressão terá quando aquecido a 60
aplicações. graus centígrados sob o mesmo volume? (R: 847,2 mm)

5 — Defina período, freqüência e comprimento de 18 — Uma lâmpada de incandescência de 125 deca-


onda de uma radiação electromagnética e diga como lúmens consome 1/2 watt por vela numa corrente de
se relacionam matemàticamente entre si estas gran- 200 volts. Qual é a resistência da lâmpada? Qual a
dezas. intensidade da corrente que a atravessa? (R: 800
ohms; 0,25 ampères).
Faculdade de Medicina
Escola Superior de Medicina Veterinária
6 — Qual deve ser o comprimento em metros de 19 — Um objecto está colocado a 3 metros de um
um fio de 0,5 mm2 de secção e de 0,004 ohms de resis- espelho esférico-côncavo resultando uma imagem real
tência específica para que, estabelecendo-se entre as e invertida situada a 30 cm do espelho. Se a imagem
suas extremidades uma diferença de potencial de passar a formar-se a 60 cm do espelho, qual deverà
220 volts, se desenvolvam nêle, em 5 minutos, 31680 ser a distância a que se colocou o objecto e qual o
pequenas calorias. Equivalente térmico do joule: aumento da referida imagem? (R: 50 cm; 12 vezes).
γ=1/4,18. (R: 1,375 m).
20 — O que entende por fenómeno de liquefacção
7 — Enuncie o princípio de Arquimedes. Como dos gases e qual a sua importância prática?
pode demonstrar-se êste princípio?
21 — Como se podem produzir os raios catódicos,
8 — Como classifica as lentes? O que entende por quais as suas propriedades e o seu principal valor
potência duma lente? Dê a definição de dioptria. prático?
Resoluções de RÓMULO DE CARVALHO

A «Gazeta de Física» publica-se para defender os interêsses dos físicos portugueses

9
Vol. I, Fasc. 1 G A Z E T A DE FÍSICA Outubro, 1946

5. EXAMES UNIVERSITÁRIOS

PONTOS DE EXAMES

F. C. L. — Física F. Q. N. — Exame final — 20 de F. C. L. — Física Geral — Exame final — 4 de Julho


Junho de 1916. de 1946.
1 — a) Indique os diferentes métodos que conhece 7—A potência consumida num circuito sem capa-
para acelerar iões e descreva um dêles. cidade (R : L=314s-1) quando se lhe aplica a tensão
b) Diga que diferença há na constituïção dos átomos eficaz V é 49,1% da potência consumida quando se
representados pelos símbolos 197 196
78 Pt e 78 Pt .
lhe aplica a tensão constante V. Calcule a frequência
da tensão alternada. R: Quando se aplica circuito a
2 — O período de semi-desintegração do Tório X tensão eficaz V tem-se:
é 3,64 dias. Preparam-se 0,82 mg dêste elemento e V V2 ⋅ R V
deseja-se saber passados 5 dias qual a massa de hélio P=VI cos φ = 2 R = 2 = .
Z R + L 4π f R(1 + L / R 4π 2f 2 )
2 2 2 2
que se formou por transmutação do Tório X no seu
elemento descendente. O número de massa do Tório X Quando em vez da tensão alternada se aplica a ten-
é 224. R: Da expressão, λT=0,693 obtém-se λ=0,190 são V contínua vem: P = 0,491 P′ em que P′ = V2/R ∴
dias-1. A quantidade I=0,32 mg de Tório X que 1
∴0,491 = donde se tira f = 50 s−1.
existe ao fim do tempo t=5 dias é dada a partir de 1 + L2 / R 2 4π 2 f 2
I =I0 e-λt. 8 — Uma ocular do tipo 1:2:4 tem a potência de
88 Th X ⎯
Atendendo ao esquema 224 ⎯
α
→ 220 4
86 Tn + 2 He deter-
5 dioptrias. A que distância da primeira lente se
mina-se que a quantidade I1=I0−I=0,50mg de Tório X deve colocar um objecto para obter uma imagem vir-
que se transmutou no elemento descendente deu lugar ao tual a 40 cm da segunda lente. R: De f 1 :d:f 2 =
aparecimento de 0,009 mg de He . d − f1 − f2
=1:2:4 e F = obtém-se f1=0,15m, f2=0,60m
f1f2
3 — Indique as aplicações à biologia e à terapia
dos neutrões e as protecções que se devem utilizar e d=0,30 m. A imagem dada pela primeira lente fun-
contra êles. ciona de objecto em relação à segunda lente. Logo de
1/p2+1/p′2=1/f2 tem-se p2=0,24m. Como p2=d−p′1→
F. C. L.—Física F. Q. N.—Exame final (2.a chamada) →p′1=d−p2=0,06 m (posição da imagem dada pela
— 4 de Julho de 1946. primeira lente). De 1/p1+1/p′1=1/f1 tira-se finalmente
p1= −0,10 m. Objecto virtual a 10 cm da primeira
4 — a) Descreva as experiências de Cockroft e lente.
Walton que constituem o primeiro exemplo de trans-
mutação provocada por partículas aceleradas artificial- 9—Um feixe de luz monocromática (λ=5000 A.U)
mente. Indique a sua importância no estudo geral incide sob o ângulo de 30° numa rêde de difracção
da Física. b) Indique como varia o número de massa (δ=12000 A. U). Determine o número de riscas que
e o número atómico de um elemento que se transmuta se formam. R: A equação da rêde de difracção:
por emissão β. Justifique a sua resposta. (1) δ(sen i+sem θ)=mλ. Atendendo a que -1 ≤ sen θ
≤1 de (1) tira-se que: -1 ≤ (mλ - δsen i)/ λ ≤1. Logo
5 — Um elemento radioactivo transmuta-se por δ(sen i -1)/λ ≤m≤ δ(1+sen i)/λ ou -12/10≤m ≤36/10
via α. O número de partículas, emitidas por hora ou ainda -1 ≤m≤ 3 . Formam-se nestas condições
por uma certa massa dêsse elemento é 240. Passados 5 riscas que correspondem a m=-1, m=0, m=1,
60 dias êste número está reduzido a 80 partículas por m=2 e m=3.
hora. Calcule o período de semi-desintegração do re-
10—a) Diga o que é a capacidade de um elemento
ferido elemento. R: Das expressões I=I0 e−λt e
de acumulador e como calcula a f. e. m. dum dínamo.
0,693t log e b) Diga e justifique porque se faz transporte de ener-
λT=0,693 obtém-se T = =38,5 dias.
log I0 − log I gia eléctrica em alta tensão e diga o que é e para que
I0 e I representam aqui intensidades em vez de quanti- serve um transformador estático. c) Esquema e fun-
dades, em virtude de o número de partículas emitidas cionamento de um rectificador de mercúrio para cor-
por uma substância radioactiva na unidade de tempo rente monofásica com rectificação total.
ser igual ao número de átomos desintegrados no mesmo
11—a) Defina coeficiente de absorção de uma
tempo e êste número proporcional à quantidade existente.
substância para um dado c.d.o. e estabeleça a rela-
6 — Indique as aplicações à biologia e à tera- ção entre esta grandeza e a espessura da substância
pia dos radio-elementos artificiais. que reduz a metade a intensidade de um feixe inci-
10
Vol. I, Fasc. 1 G A Z E T A D E FÍSICA Outubro, 1946

dente. b) Dê a expressão relativista da energia ciné- 17—a) Defina ângulo limite para duas substâncias e
tica e da quantidade de movimento de uma partícula. fluxo luminoso de um feixe. b) Dê a equação geral dos
c) Leis da emissão foto-eléctrica e equação fundamen- dióptricos e espelhos nas condições de aproximação
tal da foto-electricidade. de Gauss e estabeleça a partir dela a equação do espelho
plano. c) Um feixe de luz monocromática incide com a
12 — a) Diga quais são e o que são as amplifica-
inclinação de 45º na superfície ar-quartzo normal no
ções de um sistema óptico centrado para um dado
eixo óptico do cristal; construa os respectivos raios
ponto do eixo e dê a relação entre elas. b) Determine
refractos utilizando a construção de Huygens e os
gràficamente o feixe conjugado do feixe que incide
valores dos índices indícados na tabela das constantes.
com a inclinação de 30° num sistema óptico centrado
no ar cujos pontos cardeais se sucedem pela ordem 18—a) Diga o que é a luz de Wood e indique a cons-
HF... Descreva a construção. c) Descreva sumària- tituição do atomo 2713 Al . b) Dê a equação de De Bro-
mente as operações e cálculos a efectuar para medir
uma resistência com a ponte de Wheatstone. Esquema glie e diga como calcula a energia cinética dum
da instalação. electrão no caso mais geral. c) Indique sumàriamente
as operações e cálculos a efectuar para comparar
f. e. m. pelo processo do potenciómetro. Esquema da
F. C. L. — Física Geral— Exame final — 18 de Julho
instalação.
de 1946.
13—Um circuito compreende um gerador de f. e. m. E F. C. L. — Termodinâmica — Exame final — 13 de
de resistência interior nula em série com uma resis- Julho de 1946.
tência R e um galvanómetro de resistência inte-
rior R. Em derivação com o galvanómetro há um 19 — a) Estabeleça a expressão que relaciona os
«shunt» de resistência Rs. Calcular o poder amplifi- títulos inicial e final numa expansão adiabática
cador do «shunt» sabendo que ao retirar êste do circuito reversível de um vapor húmido. b) Dê a
é necessário substituir R por 10 R para que o galva- representação no diagrama de Clapeyron e no
nómetro mantenha o seu desvio. R: Quando se retira diagrama entrópico e indique as diferentes fases do
o «shunt» do circuito tem-se E = (Ra+10 R)I (2.ª lei de ciclo de Rankine com vapor saturado e com vapor
Kirchoff). Para o circuito com «shunt» tem-se anàloga- sôbreaquecido (ciclo de Hirn).
mente E = (R1+R)mI em que 1/R1=1/Ra +1/Rs por-
20 — a) Defina concentração, concentração mole-
tanto R1 = Ra/m visto que Rs − (Ra/m − 1). Logo
cular e título dum soluto num dos seus constituintes
(Ra/m+R) mi = (Ra+10R)I donde m=10.
e estabeleça a relação entre estas grandezas. b) In-
14—Um sistema centrado de duas lentes convergentes dique os mais importantes resultados experimentais
tem a potência de qualquer delas. Determine a posição sôbre calores específicos dos sólidos.
dos seus pontos cardiais em relação às lentes. R:
Pelos dados do problema 1/f = 1/f1=1/f2 portanto de 21—Calcular a quantidade de calor a fornecer a
1/f =1/f1 + 1/f2 − d/(f1f2) e Δ = d − f1 − f2 tem-se que: 10g de vapor de água de título 1/2 para o fazer pas-
1/f1 = 2/f1 − d/f1²; d = f1 = f2 = f e Δ = −f1 = −f2. Substi- sar a título constante, da temperatura de 40° C à
tuindo êstes valores nas relações : H1F=f1 (1 + f1/Δ); temperatura de 80° C. R: O título do vapor húmido é
H2F′ = f2(1 + f2/Δ); H1H = H1F − f e H2 H′ = H2F′ − f′ dado por x=m1/m. Para x =1/2 e m = m1+m2 =10g
vem H1F = 0; H2 F′ = 0; H1H = −f e H2H′ = −f1. de vapor húmido tem-se m1=5g de vapor sêco e
m2=5g de líquido A q. d. c. Q a fornecer à massa
15—De um par de nicois paralelos emerge um feixe m de vapor húmido para o fazer passar a título cons-
de luz ( =6000 A U) com a intensidade I0. Rodando o tante de T 1 a T 2 compõe-se de duas parcelas, uma
analizador de 45º emerge com a intensidade I1. Cal- T2

cular o percurso na água do feixe I0 para que adquira


Q1 = ∫T m1c s ' dT relativa ao vapor sêco mantendo-se
1

a intensidade I1 sabendo que o coeficiente de absorção T2


vapor saturado de T1 a T2 e a outra Q2 = ∫T m2cs dT
da água para aquela radiação é 0,0016 cm−1. 1

R: A partir de I1 = I0e-μx (lei exponencial da mantendo-se sempre líquido de saturação de T1 a T2.


absorção) e de I1 = I0cos2α (lei de Malus) obtem-se Para a água entre 0° e 200° C c1=1,02 cal/g/grau e
cos²α= e-μx donde x=433 cm. cs = cs + 796 / T cal/ g / grau). Logo Q = Q1 + Q2 =
T2 T2
16—a) Diga como calcula a f. e. m. dum alternador e = m1 ∫T (c s − 796/T )dT + m 2 c s ∫T dT = − 273,5 +
1 1

dê a fórmula de Kelvin dizendo para que serve e em +204=−69,5 cal.


que condições. b) Diga o que são um dínamo e um
magneto, o que têm de comum e o que os distingue e 22 — Calcular o rendimento global de um motor de
diga como calcula a frequência de um alternador. explosão a 4 tempos que absorve 320 calorias por
c) Diga o que é um wattímetro e qual é o seu funciona- ciclo na origem quente cuja potência é de 10 CV e que
mento e como se mede a f. e. m. dum gerador com um dá 3600 rot/min. R: Como o motor é a 4 tempos a
voltímetro. 1 ciclo <> 2 rot. O tempo correspondente a 1 ciclo,
11
Vol. I, Fasc. 1 GAZETA DE FÍSICA Outubro, 1946

nas condições do problema é t = 1/30 s. O rendimento tempo dt é dWp = dWe - dWr = aσTe4sdt - aσTr4sdt em
económico é dado ηe=Wu/Qc. Para Qc=320 que dWe é a energia emitida pela esfera, durante o
735 tempo dt e à temperatura Te da esfera e dWr a ener-
cal/ciclo e Wu= cal/ciclo vem ηe=0,18.
4,18 × 30 gia emitida pelo recinto para a esfera, no tempo dt e à
temperatura Tr do recinto que é igual à que seria emi-
23 — Uma esfera de 10 cm² de área cuja absorvi- tida pela esfera se ela se encontrasse à temperatura das
dade é 0,80 para todos os comprimentos de onda está das paredes do recinto. A potência perdida pela esfera
encerrada num recinto evacuado. Calcular a potência no instante considerado no problema é
perdida pela esfera no momento em que a sua tempe- dWp
= Pp = aσs(Te4 − Tr4 ) = 2,5 × 10 7 J/s = 2,5 watts.
ratura é 227° C e a das paredes do recinto é 27° C. dt
R : A variação de energia da esfera no intervalo de (σ = 5,735x10-5 ergs/cm/s/grau
² 4—constante de Stefan)

Resoluções de GLAPHYRA VIEIRA

PROBLEMAS PROPOSTOS
PALAVRAS PRÉVIAS

AMARO MONTEIRO

É um dos objectivos desta Revista criar e às dificuldades inerentes à subtileza do enun-


manter ou desenvolver, por todos os modos ciado se alia a complicação proveniente da
possíveis, o gôsto pelos assuntos da Física: omissão, no mesmo, das leis que relacionam
êste é então o objectivo fundamental desta entre si as grandezas a que o problema se
secção, a qual constitui por si mesma um refere.
daquêles modos possíveis. Ora nós pretendemos muitos leitores, me-
De facto é um prazer espiritual encontrar lhor, muitos colaboradores nesta secção —
a solução exacta de um problema e, daí, os provàvelmente a única que exige trabalho
numerosos cultores do género. Pedagògica- duro e permanente do leitor e para isso
mente é uma prova superior da qualidade dos entendemos que os problemas propostos de-
conhecimentos e das aptidões de uns, o desejo verão ser ou literais, ou numéricos de dados
de os desenvolver e melhorar para outros, simples que possam não conduzir a cálculos
senão todos. Contudo, ao passo que o pro- penosos e, ainda, baseados sempre em leis
blema de matemática atrai por tôda a parte vulgares a quem tenha estudos médios ou
numerosos cultores, reconhece-se neste parti- superiores de Física. Na realidade, se a cola-
cular uma certa posição de inferioridade ao boração dos físicos está sempre assegurada,
problema de física; as causas serão múltiplas, seja qual fôr a orientação seguida nesta Re-
mas reputamos essenciais as que seguidamente vista, cremos que só por esta via poderemos
apontamos. captar a dos nossos amigos matemáticos, não
Um problema de física refere-se a fenóme- lhes impondo, como ócio, a pesada tarefa de
mos físicos regidos por leis e compreende dificultosos cálculos numéricos.
grandezas e constantes físicas cujos valores A Gazeta de Física solicita dos estimados
numéricos podem conduzir os calculadores leitores que lhe enviem problemas de inte-
por vezes a operações bastante penosas. Por rêsse subordinados ao critério exposto para
outro lado as leis implícitas no enunciado serem publicados nesta secção; êstes proble-
escondem dados e hipóteses que geralmente mas deverão ser acompanhados das resolu-
estariam presentes no problema matemático ções respectivas e da indicação da fonte,
equivalente. Queremos assim significar que quando não forem originais. Publicaremos
12
Vol. I, Fasc. 1 G A Z E T A DE F Í S I C A Outubro, 1946
também para cada problema a resolução que melhoria dêste estado de coisas, publicaremos
julguemos mais clara e mais curta entre as críticas a alguns dêsses enunciados e emen-
que nos forem enviadas. daremos algumas dessas soluções: a melho-
Num dos nossos próximos números inicia- ria surgirá em futuras edições de tais
remos a publicação de problemas resolvidos, obras.
que são problemas saídos em exames de fre- Neste número da Gazeta de Física inicia-
qüência ou finais em qualquer estabelecimento remos a publicação de duas séries de proble-
de ensino, oficial ou não, desde que sejam mas, a série M baseada na Física que se
acompanhados das respectivas resoluções. estuda nos cursos médios, destinada por exem-
As obras portuguesas contendo problemas plo a alunos dos últimos anos dos cursos
de Física não são muitas; não obstante, são liceais, e a série S baseada em conhecimen-
numerosos os enunciados incorrectos, vulga- tos que só se adquirem em cursos superiores.
res os problemas de solução errada. Com o E agora mãos ao trabalho, amigo leitor; e,
objectivo exclusivo de contribuirmos para a cautela com os enunciados inocentes…

1 M — Um ascensor parte do repouso 30 m acima 1 S — Uma esfera maciça condutora com a carga
do solo, e executa o seu movimento de descida, pri- eléctrica Q em equilíbrio, de raio r, está mergulhada
meiro com a aceleração constante de 2,0 m/s² e depois no vácuo, sendo apenas actuada pela carga eléctrica
com a aceleração constante de −4,0 m/s² e atinge pontual Q′ à distância d do seu centro; a carga está
o solo com velocidade nula. Calcular o tempo de situada no vácuo também. Calcular a fôrça que actua
descida. (Mecânica por A. Ferreira). esta carga pontual.

AMARO MONTEIRO

8. DIVULGAÇÃO E VULGARIZAÇÃO
COMO SE MEDIU A CARGA DO ELECTRÃO
RÓMULO DE CARVALHO

§ 1. Os FLUIDOS ELÉCTRICOS va-lhe parte do negativo. A lã ficava electri-


zada negativamente e o vidro electrizado posi-
Até 1833 a interpretação dos fenómenos tivamente.
eléctricos assentava na existência vaga de Dêste modo e doutros semelhantes se expli-
dois fluidos que os corpos possuíam indepen- cava com sedutora facilidade a electrização
dentemente da matéria que os formava. Ti- dos corpos e as suas manifestações eléctricas
nham esses fluidos os nomes de positivo e mais elementares. Quanto à natureza dêsses
negativo e existiam normalmente em quanti- fluidos ninguém tinha idéias precisas porque
dades iguais nos vários corpos. também não as tinham àcêrca dos próprios
fenómenos. O recurso ao «fluido» foi sempre,
Se, por qualquer motivo, um corpo per- em tôda a História da Física, um recurso de
desse parte de um dêsses fluidos ficaria car- urgência para resolver interpretações difíceis.
regado com electricidade de sinal igual ao A palavra, acompanhada dum gesto vago,
do fluido que não perdera e que, por tal mo- diz tudo e não explica nada.
tivo, ficava em excesso em relação ao do outro
sinal. A causa dessa electrização poderia § 2. O ÁTOMO DE ELECTRICIDADE
ser, por exemplo, o atrito. Assim, um vidro
friccionado com um pano de lã ficava com Em 1833 surgem as primeiras razões para
excesso de fluido positivo pois a lã arranca- se admitir que os fenómenos eléctricos não
13
Vol. I, Fasc. 1 GAZETA DE FÍSICA Outubro, 1946

são devidos a quaisquer fluidos imponderáveis, drogénio era, em número aproximado, 96540
invisíveis e imateriais, mas sim a corpúsculos coulombs, e também se concluiu que essa mesma
que juntos aos corpos ou retirados deles lhes carga eléctrica libertava, no caso de o soluto
concedem as propriedades eléctricas. Esta ser um sal de prata, a massa aproximada de
nova idéia de corpúsculo não explica, na 108 gramas de prata.
verdade, o facto eléctrico mas coloca o pro- O que há nestes números de mais impres-
blema numa posição totalmente diferente da sionante é que êles (1 de hidrogénio e 108 de
anterior. Que corpúsculo é êsse? Que dimen- prata) correspondem às massas atómicas dês-
sões tem? Qual a sua massa? Como aban- ses elementos, o que nos conduz a prever a
dona os corpos? Como se introduz neles? existência de inesperada relação entre as mas-
Um aspecto particularmente interessante sas atómicas dos metais e as cargas eléctricas
desta nova idéia é o seu paralelismo com a necessárias à sua libertação.
idéia velhíssima da constituïção corpuscular O resultado surpreendente a que se chegou
da matéria. Assim como se admite que a ma- foi êste: todos os átomos-gramas de metais
téria é formada por pequeníssimas partículas, da mesma valência exigem a mesma carga
assim se passa a admitir o mesmo para a elec- eléctrica para se libertarem. Se a valência é 1
tricidade. Ao lado do átomo da matéria passa o valor dessa carga é, aproximadamente, a
a existir o átomo de electricidade, como então que apontámos: 96540 coulombs. Se a va-
se disse, dando á palavra átomo o sentido de lência fôr 2 ou 3 a carga necessária será
corpúsculo elementar. também 2 ou 3 vezes 96540 coulombs.
Foi o físico inglês Faraday que, em 1833, Se, por outro lado, soubermos, como está
nos deu a primeira suspeita da existência dum averiguado, que o átomo-grama de qualquer
corpúsculo de electricidade. Faraday, cujo elemento é constituído pelo mesmo número de
nome glorioso é bastante para envaidecer uma átomos, concluiremos que a cada átomo da
Ciência, estudava então o efeito da passagem mesma valência (e agora repare-se bem que
da corrente eléctrica através dos solutos aquo- falamos em átomos e não em átomos-gramas)
sos de substâncias minerais. O fenómeno é está associada a mesma carga eléctrica.
hoje bem conhecido e dêle resulta, em termos
sumários, a libertação de metais no eléctrodo § 3. A TEORIA CORPUSCULAR

negativo do vaso que contém o soluto. A A primeira experiência que preparou a


massa da substância assim libertada depende idéia da electricidade granular foi, como disse,
da quantidade de electricidade que atravessa realizada em 1833. As conclusões que resu-
o soluto e é-lhe directamente proporcional. mimos nas linhas anteriores foram muito me-
Verificou então Faraday que uma dada quan- nos rápidas do que se poderia imaginar.
tidade de electricidade ao atravessar um so- Levaram algumas dezenas de anos a estabe-
luto dum ácido mineral liberta, no polo ne- lecer-se, foram apoiadas por uns e criticadas
gativo do vaso, sempre a mesma massa de por outros, até que se firmaram em segura
hidrogénio qualquer que seja o ácido que se base. Só bem no fim do século XIX é que se
utilize. Anàlogamente, se uma dada carga admite, sem controvérsia, a existência dum
eléctrica atravessa o soluto dum sal dum me- corpúsculo eléctrico relacionado com a exis-
tal M, a massa metálica libertada é sempre tência do átomo. Foi só então, em 1891,
a mesma qualquer que seja o sal de que se que Stoney propôs que êsse corpúsculo ti-
trate. Isto permite concluir imediatamente vesse um nome próprio e o nome proposto
que há sempre uma certa quantidade de elec- e aceite foi o de Electrão.
tricidade relacionada, seja como fôr, com as vá- Encontramo-nos assim em presença de uma
rias massas dos vários metais. unidade natural de carga eléctrica, o electrão.
Concluiu-se da experiência que a carga eléc- A sua definição, na época, apontada, será:
trica necessária para libertar 1 grama de hi- a quantidade de electricidade que deverá atra-
14
Vol. I, Fasc. 1 GAZETA DE FÍSICA Outubro, 1946

vessar um soluto para libertar no eléctrodo dos seus electrões (que são negativos) fica
negativo um átomo de hidrogénio ou um electrizado positivamente e passa a denomi-
átomo de qualquer outro elemento monova- nar-se um ião positivo; se, pelo contrário, um
lente. átomo capta um ou mais electrões fica elec-
Terminou o século XIX convencido da exis- trizado negativamente e deixa de ser um
tência de corpúsculos materiais (moléculas e átomo para ser um ião negativo.
átomos) e de grãos de electricidade (os elec-
trões). Com estes se pretendiam explicar as § 4. O FUNDAMENTO DO PROCESSO
propriedades eléctricas da matéria. DA MEDIDA DIRECTA DA CARGA DO ELECTRÃO
Começa, porém, o século XX e logo, no seu
alvorecer, se alarga de maneira imprevista e Se qualquer porção de matéria é constituída
soberba o conhecimento da estrutura dos cor- por corpúsculos eléctricos será possível isolar
pos. O olhar arguto dos cientistas devassa a um ou mais dêles e medir-lhes a carga?
intimidade insuspeitada do mundo dos corpús- Qual o homem que não estaca de assombro
culos. O átomo deixa de ser a partícula una perante o arrôjo desta pregunta? Pois bem.
e indivisível para ser um pequeno universo A pregunta foi feita e a resposta foi dada. Iso-
de partículas várias que gravitam sem des- lou-se um electrão e mediu-se-lhe a carga eléc-
canso segundo leis novas onde o arrôjo da trica, directamente, apesar de se tratar duma
concepção dos físicos é posto à prova mesmo partícula cujas dimensões vão muito abaixo
sob a pena de destruirem muito saber ante- das dimensões do átomo as quais já por si
riormente acumulado. são tão insignificantes que não podemos conce-
Dentro da nova idéia o grão de electrici- bê-las.
dade passa a considerar-se componente de O gigante da Física que mediu directamente
tudo quanto existe, desde o muitíssimo pe- a carga do electrão foi Robert-Andrews Mil-
queno até o muitíssimo grande. Agora tudo likan no Laboratório Norman Bridge do
são grãos de electricidade. Todos os átomos Instituto da Califórnia em Pasadena. Êste
de todos os elementos são formados por par- trabalho, como todos os trabalhos científicos,
tículas que são grãos de electricidade, umas não surgiu esporàdicamente da mão e do
positivas e outras negativas, em número maior cérebro dum só homem. Outros o antecede-
ou menor, consoante a natureza do átomo. ram e outros o acompanharam. Com Milli-
Tôdas essas cargas, porém, em cada átomo, kan, porém, o problema foi magistralmente
compensam os seus efeitos e o conjunto apre- resolvido, com requintes técnicos admiráveis
senta-se-nos neutro como na velha teoria dos e ao fim de muitos anos de esforços continua-
dois fluidos quando ambos se compensavam. mente aperfeiçoados.
Se, eventualmente, entrar, ou sair dêsse pe- Eis o fundamento do processo da medida
queno universo, um ou mais grãos de electri- directa da carga do electrão.
cidade, eis que o conjunto se desequilibra e Imaginemos uma chuva de gotas muito
aí o temos electrizado positivamente ou nega- miudinhas que caem verticalmente e serena-
tivamente, conforme a mecânica do fenómeno. mente. Suponhamos que é possível separar
Assim se passam a explicar, com maior ou uma só gota de tôdas as outras e que, de
menor cópia de pormenores, as propriedades qualquer modo, a electrizamos. Electrizá-la
eléctricas da matéria. O termo electrão man- significa, dentro da teoria que apresentámos,
tém-se através das novas concepções mas como que obrigamos a gota a libertar um ou mais
agora há corpúsculos eléctricos de duas espé- electrões dos seus átomos ou então que a força-
cies reserva-se o nome para o corpúsculo ne- mos a captar um ou mais electrões existentes no
gativo. A linguagem própria passa então a espaço onde ela cai. No primeiro caso a gota
ser esta: se um átomo dum elemento perde, ficaria electrizada positivamente e no segundo
não importa como nem porquê, um ou mais caso, negativamente.
15
Vol. I , Fasc. 1 GAZETA DE FÍSICA Outubro, 1946

Imaginemos agora que esta gota electrizada câmara C o qual se espalhava em gotas finís-
é obrigada a prosseguir a sua queda no inter- simas cujos raios eram da ordem de um
valo entre dois pratos metálicos colocados milésimo de milímetro. Tóda a câmara C
horizontalmente e a pequena distância um do estava contida noutro recipiente (que a figura
outro. Se, durante essa queda, ligarmos os não representa) e completamente envolvido
pratos a um circuito eléctrico, o movimento por 40 litros de um óleo cujo fim era o de
da queda livre da gota será imediatamente manter constante a temperatura da câmara,
perturbado. Se a gota estiver electrizada condição essencial para o bom êxito da expe-
positivamente será atraída para o prato nega- riência.
tivo e se o estiver negativamente será atraída 2.°. Isolamento duma gota—No meio do
para o positivo. Esta atracção, dirigida no prato circular p1 fizera-se um orifício tão fino
mesmo sentido ou no sentido oposto da queda como se fôsse aberto por uma agulha fina que
da gota, modificará a velocidade do movi- o atravessasse. Das várias gotas que caíam
mento e será então possível conhecer o valor lentamente na câmara C, havia uma, de
da carga eléctrica a partir do conhecimento quando em quando, que caía na direcção do
da mudança que a velocidade sofreu. orifício e ficava na situação desejada entre os
Foi esta extraordinária concepção que pratos p 1 e p 2 . Mas... como ver esta gota?
Millikan realizou com surprendente êxito. Da seguinte maneira. Os dois pratos p1 e
Vejamos como. p2, que eram discos de latão com 22 cm de
diâmetro, estavam situados a 16 mm um do
§ 5. A TÉCNICA DO PROCESSO outro e rodeados por um anel (a, α) de ebo-
Em 1916, depois de alguns anos de expe- nite que fechava completamente o intervalo
riências cada vez mais cuidadosas e minu- entre os pratos. Nêsse anel abriam-se três
ciosas, Millikan terminou o seu trabalho. pequenas janelas em posições convenientes.
A figura que se segue mostra, muito sumà (A figura só representa duas: a, a). Por

riamente, o dispositivo utilizado. Nela está uma delas (suponhamos a da esquerda da


indicado só o que é essencial à compreensão figura) entrava um feixe intenso de luz que
do assunto mas, na realidade, a aparelhagem ia iluminar a gota que porventura entrasse no
era muito mais complicada. Sigamos a ordem intervalo dos pratos. Por uma das outras
indicada anteriormente. janelas (suponhamos a que não está indicada
1.°. Formação da chuva de gotas miudi- na figura) o observador via a gota iluminada
nhas — O tubo P da figura representa um como se fôsse uma estrêla brilhante no meio
pulverizador que era accionado por pressão obscuro dos pratos. A luz era produzida por
de ar muito bem sêco e liberto de tôdas as uma lâmpada de arco e atravessava primeira-
poeiras. Por meio dêle lançava-se azeite na mente uma tina T de 80 cm de comprimento,
16
Vol. I, Fasc. 1 G A Z E T A D E FÍSICA Outubro, 1946

cheia de água, e depois outra, t, com soluto § 6. OS CÁLCULOS


de cloreto cúprico, ambas com o fim de absor-
Sabemos que fôrças de intensidades diferen-
ver os raios caloríficos da luz. Pela segunda
tes aplicadas ao mesmo corpo lhe concedem ve-
janela indicada observava-se a gota por meio
locidades proporcionais aos valores das fôrças.
de uma luneta de curto foco e cuja ocular
Seja mg o peso da gota; v1 a sua velocidade
continha uma escala que permitia medir a
em queda livre; Fe a fôrça eléctrica que so-
velocidade com que a gota caía.
licita a gota, sendo e a sua carga e F o valor
3.°. Electrização da gota — A electrização do campo criado pelo circuito dos pratos no
da gota era feita por meio dos raios X emi- intervalo entre êles; v2 a velocidade do mo-
tidos por uma ampola. É sabido que os vimento quando a gota está sujeita, simultâ-
raios X tornam os gases condutores pois des- neamente, ao seu peso e ao campo eléctrico.
carregam um electroscópio colocado no am- Teremos:
v1 mg
biente que os raios influenciam. Quando a = ⋅
radiação atravessa o meio considerado expulsa v 2 Fe ± mg
electrões dos átomos que formam êsse meio. Os sinais + e − referem-se aos casos das
Se a radiação incidir sôbre a gota que cai, duas fôrças actuantes terem o mesmo sentido
ela torna-se positiva devido à perda de elec- ou sentidos contrários.
trões, e a sua carga será então igual e de Quanto ao valor e que se pretende conhe-
sinal contrário à carga que perdeu. Saber a cer, êle pode naturalmente ser a carga de 1
carga que possue é o mesmo que saber a carga electrão ou de mais do que 1. Contudo, em
que perdeu. No dispositivo de Millikan a muitíssimas experiências realizadas por Milli-
radiação produzida em X incidia no intervalo kan, foi sempre encontrado um valor mínimo
entre os pratos passando através da terceira de e, o qual valor foi de 4,774×10−10 U. Es. Q.
,janela a (à direita da figura). (unidades electrostáticas de quantidade de
electricidade).
4.°. Circuito dos pratos — Os pratos p 1 e A massa m da gota, que é esférica, cal-
p2, faziam parte dum circuito alimentado por cula-se pela expresssão m=4/3πa3σ em que a
uma bateria de 10 mil volts. Um comutador é o raio da gota e σ a sua massa específica.
apropriado permitia dar aos pratos o sinal que Por seu turno o raio da gota foi calculado a
mais conviesse e também pô-los em curto- partir da fórmula de Stokes que dá a velo-
circuito. Millikan conseguia observar a queda cidade da queda duma esfera polida e rígida
da gota electrizada durante minutos sucessivos através dum meio uniforme. Ela diz que:
sem sair do pequeno intervalo entre os pratos, 2 ga 2
ora fazendo-a subir, ora fazendo-a descer, ora v1 = (σ − ρ )
9 η
fazendo-a parar.
em que η é a viscosidade do ar e ρ a sua massa
5.°. A medida das velocidades da gota era especifica.
feita por meio da luneta do observador. Na Tôdas as constantes foram determinadas
ocular da luneta estavam traçados 3 fios equi- com extraordinário rigor o que exigiu pacien-
distantes cujas distâncias eram rigorosamente tes e delicadíssimos trabalhos á margem do
conhecidas. Um cronógrafo especial gravava os fim principal da obra.
tempos das subidas e descidas com a preci- Página imortal da Física é esta e padrão
são de centésimos de segundo. glorioso de quanto pode o génio dos homens.

RÓMULO DE CARVALHO
PROF. DO LICEU CAMÕES

A «Gazeta de Física» luta por um curs o independente de Física

17
Vol. I, Fasc. 1 GAZETA DE FÍSICA Outubro, 1946

9. HISTÓRIA E ANTOLOGIA
ENGENHARIA ATÓMICA?

THEODORE VON KÁRMÁN

Pela terceira vez na nossa vida (o «nossa» maravilhosa técnica hoje vulgarmente desi-
referindo-se aos veteranos da engenharia) os gnada por arte do radar. Os factos físicos
físicos abriram-nos novos caminhos de activi- eram conhecidos muito antes do começo desta
dade extremamente importantes, e o enge- guerra. No entanto, foi durante ela que os
nheiro não se encontrou apto a enfrentar tal físicos, com o seu conhecimento profundo da
tarefa. electrónica, isto é, da mecânica das partículas
Em 1887, o físico Heinrich Hertz descobriu eléctricas, conseguiram resolver o duplo pro-
a «telegrafia sem fios». Na verdade êle con- blema de engenharia de produzir e orientar
firmou experimentalmente a existência de on- as micro-ondas.
das electromagnéticas que qualquer podia E agora, parece estarmos no patamar da
reconhecer nas equações de Maxwell do nova era atómica. Não sei se isto é ou não
campo electromagnético, isto é, qualquer que verdade, mas certamente teremos «engenharia
fôsse capaz de ler uma equação. atómica» nos domínios da energia e dos trans-
Levou ainda algumas décadas até que a portes. Estaremos nós preparados para os
rádio se tornasse um ramo de engenharia e problemas que isto envolve?
que se tomassem medidas adequadas para Eu pergunto: dão-se hoje ao futuro enge-
incorporar os fundamentos da teoria do campo nheiro conhecimentos fundamentais suficientes
no curriculum do engenheiro. nas questões básicas da estrutura da matéria,
À descoberta das ondas electromagnéticas da natureza da energia e da relação entre
de grande comprimento de onda seguiu-se, matéria e energia de modo a que êle fique
pouco depois, a descoberta dos raios X, isto apto a pensar inteligentemente neste novo
é, de ondas electromagnéticas de muito pe- campo, a fazer o bom juízo sabre as suas pos-
queno comprimento de onda e de freqüência sibilidades e métodos? Haverá alguma falha
muito além das correspondentes ao espectro na formação do nosso engenheiro? Infeliz-
da luz visível. Deve dizer-se, em abono da mente há.
engenharia, que esta se aproveitou da técnica Em primeiro lugar eu creio que temos certa
dos raios X em vários ramos da indústria, e tendência a restringir o nosso ensino aos
que o estudo e construção de montagens de conhecimentos científicos de aplicação ime-
raios X foram reconhecidos como uma das diata. Esquecemo-nos muitas vezes que quási
suas secções importantes. tôdas as descobertas da física e da química
Do mesmo modo, a técnica do infra-verme- podem ter aplicação na engenharia. Na ver-
lho, isto é, a técnica de radiações com um dade, esta representa a conquista da natureza,
comprimento de onda superior ao da luz visí- isto é, da matéria e energia, de modo a levar
vel, mas muito inferior ao das ondas da rádio, confôrto à humanidade, e, portanto, um en-
foi com justiça considerada como um problema genheiro nunca pode saber de mais sôbre a
de engenharia. constituïção interna dessa matéria cujos esta-
Contudo, no intervalo que medeia entre as dos caprichosos êle combate e sôbre as leis
duas guerras mundiais, os físicos fizeram gran- dessa energia que êle quere desvendar e do-
des esforços para diminuir a lacuna entre on- minar.
das hertzianas e infra-vermelhas. Conseguiram Em segundo lugar, nós subestimamos o
produzir micro-ondas que tornaram possível a interêsse dos nossos estudantes pela «filoso-
18
Vol. I, Fasc. 1 GAZETA DE FÍSICA Outubro, 1946

fia da natureza». Temos relutância em apre- para seguir o desenvolvimento da biologia,


sentar os fundamentos da física e da química que tem produzido nos últimos tempos des-
como uma ciência viva, cheia de pontos de cobertas quási tão espectaculares como a
interrogação e conceitos variantes. E porque física.
razão uma combustão vulgar é um problema Estou convencido que tanto o conhecimento
de engenharia e uma reacção nuclear um das origens mais remotas como o das limita-
mistério de alquimia moderna? ções dos princípios não entrava o indivíduo
Em terceiro lugar achamos mais importante quando da sua aplicação prática; na verdade
fornecer um amontoado de conhecimentos às um conhecimento real torna a aplicação mais
futuras gerações de engenheiros do que um fácil e segura. E, de facto, os estudantes de
ensino tendo por fim uma compreensão real física não são nem mais nem menos inteligen-
dos fenómenos da natureza. Tentamos dar tes que os de engenharia.
aos engenheiros um treino tal que o patrão Tenho-me afastado do assunto indicado no
os possa utilizar quási imediatamente após a título. Com o pouco que sei e com o pouco
licenciatura. Um dos meus antigos alunos que li, creio firmemente que os problemas in-
estava colocado como professor numa conhe- cluídos no desenvolvimento do poder atómico,
cida escola de engenharia. Sugeriu algumas quer para fins estáticos quer para fins de
modificações no programa de hidráulica, no transporte, são de primeira importância na
sentido da moderna dinâmica dos fluidos. engenharia e mecânica aplicada. Por outras
O seu superior preguntou-lhe qual a vantagem palavras, o quadro dos processos nucleares
em ensinar os resultados de investigadores, obtido pelos físicos tem tanto de verdade, se
que êle no entanto concedia terem obtido tal coisa existe, que de futuro, as observações
sucessos interessantes na hidrodinâmica. Ne- sistemáticas e os cálculos terão por fim solu-
nhum dêsses homens, dizia êle, seria capaz ções de ordem prática. Certamente hão-de
de desenhar um sistema de esgotos. Feliz- surgir problemas atordoantes de engenharia,
mente, os patrões mais inteligentes começam como o da resistência de materiais a tempe-
a sentir bem que o critério mais importante raturas e corrosões extremas, e aplicações
para um noviço de engenharia não é o da difíceis dos nossos conhecimentos sôbre difu-
aplicação imediata. Já apreciam uma com- são e transferência de calor. No entanto, e
preensão lógica dos fundamentos e não pre- pode parecer paradoxal, creio que os proces-
tendem que as escolas lhes treinem o tipo de sos que a engenharia atómica envolve são
engenheiro prático, o qual — segundo o bon menos complexos que, por exemplo, os da
mot dum inglês de renome, também enge- combustão dos combustíveis sólidos e líquidos
nheiro, — perpetua os êrros dos seus prede- usuais; menos complexos no sentido de que,
cessores. considerações simples e alguns cálculos teó-
Em quarto lugar, não quero de nenhum ricos podem levar a uma maior aproximação
modo fazer um cientista de cada engenheiro, da realidade que no caso das reacções mole-
ou ainda menos um físico nuclear. No entanto culares. Que mais pode agradar a um enge-
tentaremos dar-lhe uma educação científica nheiro científico?
que lhe permita seguir o progresso da ciên-
cia; talqualmente como eu creio que todo o Passagens extraídas do Vol. 17, n. 1 do Journal
médico deve ter uma preparação suficiente of Applied Physics.
TRAD. TEODOLINDA FREIRE

A indústria nacional só pode desenvolver-se com o auxílio


de físicos e químicos portugueses
19
VoI. I , Fasc.1 GAZETA DE FÍSICA Outubro, 1946

G ABRIEL LIPPMANN

MARIA HELENA BLANC DE SOUSA

Fêz no dia 31 de Julho, vinte e cinco anos fôr revelada pelos modos usuais e depois ilu-
que morreu Lippmann. minada normalmente com luz branca, notar-
Nascido em 1845 no Luxemburgo, foi um se-à a chapa corada ùnicamente pela radiação
dos físicos mais brilhantes do seu tempo. de c. d. o. λ′, isto é, pela radiação que deu
Foi professor de Física Experimental e de origem aos seus planos nodais e ventrais, o
Física Matemática na Faculdade de Ciências que se explica pelo facto desta radiação ser
de Paris e desempenhou até à morte o cargo reforçada nos estratos equidistantes de λ′/2,
de Director do Laboratório de Investigação ao passo que, as outras radiações de c. d. o.
na Sorbonne. diferentes de λ′, são enfraquecidas.
Dedicou-se a vários estudos sôbre Astro- Generalizando, se a chapa fôr impressio-
nomia, mas a sua maior contribuição foi para o nada por um feixe de luz heterocromática,
estudo da Física, deixando o seu nome ligado cada ponto, ao ser iluminado normalmente
para sempre ao método da fotografia a côres. com luz branca, reflete as radiações corres-
Êste método baseia-se no facto de, na re- pondentes às que a impressionaram.
flexão da luz por um espelho, os raios inci- Em 1893, Lippmann apresentava pela pri-
dente e reflectido formarem um sistema de meira vez à Academia das Sciencias de Paris
ondas estacionárias. Se o espelho estiver fotografias nas quais as côres eram reprodu-
coberto por uma camada espêssa, contínua e zidas com perfeito ortocromatismo.
sem grãos de cloreto de prata, e o iluminarmos Ä
com um feixe de luz monocromática, formam- Foi eleito membro da «Royal Society» em
-se na camada sensível, estratos equidistantes 1908 e no mesmo ano recebia o prémio Nobel.
de prata, correspondentes aos planos ventrais. Morreu com 76 anos, na viagem de regresso
A distância entre êsses estratos é de λ′/2, do Canadá e dos E. U. da América, onde
sendo λ′ o c. d. o. da radiação no meio cons- tinha ido como membro duma missão fran-
tituído pela camada sensível; se esta chapa cesa.
M. H. BLANC DE SOUSA

10. QUÍMICA
ORIGEM E OBJECTIVO DESTA SECÇÃO
MARIETA DA SILVEIRA
Desde há muito que, no nosso País, se faz última, criando na Gazeta de Física uma
sentir a falta de uma revista em que fôsse secção de Química. Esta secção terá pois,
possível tratar vários problemas, respeitantes como objectivo, preencher, embora duma ma-
tanto ao ensino secundário como ao ensino neira muito incompleta, aquela lacuna, até
superior da Química, e em que, além disso, que seja possível a criação duma revista de-
se pudesse procurar levar ao conhecimento dicada exclusivamente aos problemas da Quí-
dos estudiosos portugueses os progressos mica. Assim, numa única secção, procurare-
mais importantes realizados no campo da mos, tanto quanto possível, abordar todos os
Química Moderna. problemas, tanto os pedagógicos, como os
Na impossibilidade de criar simultânea- científicos e industriais, que, de qualquer modo,
mente uma Gazeta de Física e uma Gazeta de possam interessar àquêles que, em Portugal,
Química, procurou remediar-se a falta desta se dedicam ao ensino ou ao estudo da Química.
MARIETA DA SILVEIRA

20
Vol. I, Fasc. 1 GAZETA DE FÍSICA Outubro, 1946

NOMENCLATURA QUÍMICA
ALICE MAIA MAGALHÃES

É bem conhecida a importância da nomen- riormente: carbonato de sódio e potássio,


clatura em todos os ramos da Química. CO3NaK; fosfato de amónio e magnésio,
Torna-se no entanto indispensável, para PO4MgNH4.
que possa haver perfeita compreensão e não Havendo vários constituintes electro-nega-
se suscitem dúvidas, uniformizar as regras da tivos e um só electro-positivo, figura igual-
nomenclatura, de modo que os nomes dos mente êste no fim, e os outros aparecem por
compostos obedeçam à dupla condição da ordem de propriedades electro-negativas de-
clareza e simplicidade. crescentes: sulfo-arsenieto de ferro, SAsFe;
Tal é o papel da Comissão de Reforma da fluo-cloreto de chumbo, FClPb.
Nomenclatura Química, da União Internacio- Note-se que «quando num radical ácido o
nal de Química. enxofre toma o lugar do oxigénio, deve desi-
Por outro lado, convém divulgar entre nós gnar-se por tio e não por sulfo»: tiocianato
as regras adoptadas universalmente, a fim de de potássio, SCNK; tiosulfato de sódio,
evitar, na designação dos compostos, diver- S2O3Na2.
gências sempre prejudiciais. Sempre que o metal apresente, em geral,
Por êsse motivo, procurámos condensar uma valência constante, torna-se desnecessá-
algumas das principais regras apresentadas ria a indicação desta.
no relatório da citada comissão, referentes à Mas a caracterização das proporções dos
Química Inorgânica.(1) constituintes, num composto químico, pode
ainda ser feita de outra maneira, indicando a
Ä
composição estequiométrica, por meio de pre-
fixos de origem grega aplicados aos consti-
1. Tanto no nome do composto como na
tuintes: dissulfureto de ferro, S2Fe; diclo-
respectiva fórmula, o elemento ou radical
reto de cobre Cl2Cu.
electro-negativo, deve figurar em primeiro
Êste processo é cómodo, por exemplo para
lugar.
os compostos oxigenados de azoto: ON2, mo-
2. A valência electro-química do elemento
nóxido diazótico ou óxido diazótico; ON,
positivo indica-se por meio de algarismos
óxido de azoto; O 2 N, dióxido de azoto;
romanos, escritos entre parêntesis, a seguir
O 3 N 2 , trióxido diazótico; O4N2, tetróxido
ao nome do elemento. Exemplos: cloreto de
diazótico; O5N2, pentóxido diazótico.
cobalto (II), Cl 2 Co; óxido de ferro (II),
Dêstes, o quarto e o sexto continuam a ser
OFe; óxido de ferro (III-II), O 4 Fe 3 , sulfato
de alumínio (III) (SO4)3Al2. designados, sem inconveniente, pelos anterio-
Esta designação da valência, muito cómoda res nomes de anidrido azotoso e anidrido
e bem clara, procurou eliminar os inconve- azótico.
nientes da caracterização daquela grandeza, Sais ácidos. Os sais contendo hidrogénio
por exemplo pelas terminações o, i (ferro, ácido designam-se por hidrogeno-sais, e nos
ferri), e oso, ico (cuproso, cúprico). seus nomes indica-se o número de átomos de
Havendo mais do que um constituinte hidrogénio. Exemplos: hidrogeno-sulfato de
electro-positivo e um electro-negativo, êste sódio, SO4HNa; hidrogeno-fosfato dissódico,
continua a figurar no princípio, como ante- PO4HNa2; dihidrogeno-fosfato monossódico,
PO4H2Na.
(1) Bulletin de la Société Chimique de France — Sais básicos. A existência, nestes sais, de
t. 8. pg. 814, 1941. grupos oxidrílicos, confere-lhes a categoria de

21
Vol. I, Fasc. 1 GAZETA DE FÍSICA Outubro, 1946

hidroxi-sais. Exemplo: hidroxi-cloreto de dá-se a terminação ato ao nome do metal


cádmio, (OH)ClCd. central, precedendo-o da designação dos radi-
Se o metal está ligado simultâneamente a cais existentes e respectivo número, e seguin-
um radical ácido e a átomos de oxigénio, temos do-o do nome do catião. Indica-se também a
os óxi-sais. Exemplo: oxicloreto de bis- valência do metal central (e a do catião, se fôr
muto, OClBi. necessário). Exemplos: hexaciano-ferrato (II)
de potássio(1) , [Fe(CN) 6 ]K 4, hexaciano-fer-
Complexos rato (III) de potássio, [Fe(CN) 6 ]K 3 ; exani-
tro cobaltato (III) de potássio, [Co(NO2)6]K3;
dinitro oxalato diamina cobaltato (III) de
1.° Catiões complexos. Indica-se o anião potássio, [Co(NO2)2(C2O4) (NH3)2]K.
(cloreto de. . . , sulfato de. . .), a seguir os
Tal como anteriormente, também a nomen-
radicais que fazem parte do complexo, em
clatura dêstes compostos pode, em muitos
ligação coordenada e os grupos moleculares
casos, ser feita como segue: Para o primeiro,
nas mesmas condições, e finalmente o nome
hexaciano ferrato tetrapotássico, etc.
do metal, com a designação da valência, como
anteriormente. Tratando-se de ácidos de anião complexo,
temos: ácido hexacloro platínico (IV),
Os principais radicais naquelas condições
[PtCl 6 ]H 2 , ácido hexaciano férrico (II),
são (pela ordem em que figuram nas fór-
[Fe(CN) 6 ]H 4 ácido hexaciano férrico (III),
mulas): Cl (cloro) e Br (bromo), CN (ciano),
[Fe(CN)6]H3.
OCN (cianato), SCN (tiocianato), SO4 (sul-
fato), NO 2 (nitro), ONO (nitrito), C 2 O 4 , (oxa- 3.° Catião e anião complexos. Para êsse
lato) e OH (hidroxo); os agrupamentos caso, não é preciso mais do que conjugar as
moleculares, de valência nula, são: OH2 regras apresentadas para os casos isolados:
(aquo), NH 3 (amina), etc. Exemplos: clo- hexanitro cobaltato (III) de hexamina cobalto
reto de dihidroxo tetramina cobalto (III), (III), [Co(NO2)6][(NH3)6Co].
Cl[(OH) 2 (NH 3 ) 4 Co]; sulfato de cloro pen-
taquo crómio (III), SO 4 [Cl(OH 2 ) 5 Cr]; 4.° Complexos não ionizáveis. A sua no-
brometo de dicloro diamina platina (IV), menclatura faz-se por um processo semelhante
Br 2 [Cl 2 (NH 3 ) 4 Pt]; cloreto de dibromo dia- aos anteriores: tritiocianato triamina crómio
mina platina (IV), Cl2[Br2(NH 3 ) 2 Pt]. (III), [(SCN)3(NH3)3Cr].
Por vezes pode substituir-se a indicação da
valência pelo emprêgo de prefixos aplicados ALICE MAIA MAGALHÃES
ao nome do anião: tricloreto de hexamina
cobalto C13[(NH3)6Co]. (1)Parece-nos mais lógico fazer figurar na fórmula
o metal do anião em último lugar, ao contrário do
2.° Aniões complexos. Tratando-se de sais, que apresenta o Bulletin de onde retirámos estas notas.

PONTOS DE EXAMES DE APTIDÃO


F. C. L. — Agôsto de 1946. tifique o modo como acha a relação aproximada das
quantidades de calor absorvidas. Dados: Al=27
1 — Um soluto aquoso congela a −0,1°C. A que Fe=56. R: Das expressões gerais Q=mc∆t e cA = 6,4
temperatura ferve à pressão normal? Dados: Kc= e, por serem iguais as massas e as variações de tempe-
=1850; K e =520. R: De ∆t=K c /P e ∆t′=K e n/P ratura, resulta :
vem ∆t/∆t′=Kc/Ke ou ∆t′=0,028°C. logo t=100,028° C.
QAl/QFe = CAl/CFe = AFe /AAl = 56/27≈2.
2 — Aquecemos simultâneamente dois recipientes,
um de alumínio e outro de ferro, de pêso igual. Jus- 3 —1 cm ³ dum óleo, de densidade 0,9, contém
22
Vol. I, Fasc. 1 GAZETA DE F Í S I C A Outubro, 1946

0,005 g dum ácido. Qual é a sua acidês, por cento e 5 — Justifique a lei de Dalton com o sesquióxido
por litro? R: 0,56 %; 5 g/l. de ferro e o óxido salino. R: A razão das massas de
4 — Calcule a densidade e o pêso de 1 litro de ozono. ferro combinadas com a massa constante 16 g de oxi-
Justifique o cálculo. R: dar =1,67; p=2,143 g. génio é m/n = 8/9.

PROBLEMAS DE EXAMES UNIVERSITÁRIOS


F. C. L. — Curso Geral de Química e Curso de Química 6 — Sabendo que 10 mg dum elemento radioactivo
F. Q. N. — Julho de 1946. perderam, em 10 anos, um décimo da sua actividade,
a que conclusão chega? R: Da expressão q=q0e−λt
1 — Esquematize uma síntese realizável do com-
conclui-se que: λ=log (q0/q):(t.log e)=0,0105 anos−1
COC6H5 , a partir do e das expressões λT=0,693 e θ=1/λ tira-se: λ=66
posto de fórmula C 6 H 4
OH anos e θ=95 anos .
benzeno. R : a) Síntese do fenol, a partir do benzeno:
7 — Desidrata-se o carbonato de amónio. O produto
SO4H2 fumante fusão com OHNa sólido formado é tratado por uma solução alcalina de
C6H6 ⎯⎯⎯⎯⎯⎯⎯→ C6H5SO3H ⎯⎯⎯⎯⎯⎯⎯→
hipobromito de sódio, libertando-se 125 cm3 dum gás,
SO4H2 dil.
⎯⎯→ C6H5ONa ⎯⎯⎯⎯⎯→ C6H5OH. nas condições p. t. n. Pede-se: a) Os esquemas das
reacções; b) o nome do gás libertado e o do produto
b) Formação do cloreto de benzoílo:
sólido; c) o pêso de carbonato utilizado.
C13Al anidro R: a) — CO3(NH4)2 ⎯⎯⎯→ CO(NH2)2 + 2OH2
C6H6 + C12CO ⎯⎯⎯⎯⎯→ C6H5COCl + ClH.
CO(NH2)2 + 3BrONa + 2OHNa ⎯⎯⎯→
c) Condensação, segundo Friedel e Crafts, do cloreto ⎯⎯→ CO3Na2 + 3BrNa + N2 + 3OH2
de benzoílo com o fenol: b) Azoto e ureia; c) 0,535 g.
C13Al anidro
C6H5COCl + C6H5OH ⎯⎯⎯⎯⎯⎯⎯⎯⎯⎯⎯⎯→
8 — 0,8914 g do cloroplatinato duma amina orgânica,
⎯⎯→C6H4 COC6H5 com 68,19 % de carbono, 7,38 % de hidrogénio e
+ ClH.
OH 11,42 % de azoto, deixam um resíduo de 0,2652 g.
Tratados, pelo método de Zeisel, 0,5 g produzem
2 — Indique, esquemàticamente, como faria a síntese
0,9544 g de iodeto de prata. 0,123 g deslocam, no
da benzamida, a partir do etanol. R: CH3CH2OH→ aparelho de V. Meyer, 22,4 cm3 de ar (p. t. n.). Indi-
O2 Cl5P OH alc. car a fórmula racional do composto.
⎯→ CH3CHO ⎯⎯→ CH3CHCl2 ⎯⎯⎯→ CH≡CH
OCH3
polimeriz. Cl2CO NH3 R: C 6 H 4
⎯⎯⎯⎯→ C6H6 ⎯⎯→ C6H5COCl ⎯→ C6H5CONH2. NH2

3 — Esquema da síntese dum propanoloico a partir F. C. L. — Análise Química, 1. a parte — Julho de 1946
O2 CNH
do etanol. R : CH3CH2OH ⎯⎯→ CH3CHO ⎯⎯⎯→ 9 — 50 cm3 duma solução de carbonato e de bicar-
OH2 + ClH bonato de sódio foram tratados por ClH, N/40, em
⎯→ CH3CHOHCN 2 ⎯⎯⎯⎯⎯→ CH3CHOHCOOH. presença de fenolftaleína. Gastaram-se 37,8 cm3 do
soluto ácido, até viragem do indicador. Adicionaram-se
4 — Esquematize a síntese da glicilglicina, a par- 2 gotas de heliantina e, para obter de novo a viragem,
Cl2 OHK alc. gastaram-se 27,9 cm3 de ClH, N/20. Indicar a com-
tir do etano. R: CH3CH3 ⎯→ CH3CH2Cl ⎯⎯⎯⎯→ posição da mistura. R: O 1.° ensaio corresponde à
SO4H2 Cl2 transformação: CO3Na2 + ClH → CO3HNa + ClNa,
→ CH3CH2OH ⎯⎯→ CH2=CH2 ⎯→ CH2ClCH2Cl →
donde se conclui que a quantidade de CO3Na2 em 50 cm3
OHK alc. O2 NH3
⎯⎯⎯→ CH2OHCH 2 ⎯⎯→ CH2OHCOOH ⎯⎯→ de solução é x=0,100 g.
⎯⎯→ CH2NH2COOH. O 2.° ensaio corresponde a

2CH2NH2COOH ⎯→ CH2NH2CONHCH2COOH + OH2 CO3HNa + ClH → CO2+OH2 + ClNa.

5 — Determinar os valores de PH e Ka dum mono- O bicarbonato que intervém nesta reacção, é o que já
ácido fraco, cujo soluto aquoso 0,1 N congela a existia na solução, mais o que proveio da transforma-
−0,279° C. (constante crioscópica da água: 1850). ção anterior. Considerando ClH, N/40, a quantidade
R: Atendendo à lei de Raoult, ∆t=K n [1 + α(n1−1)]/P de CO3HNa, existente em 50 cm3 da solução, corres-
tem-se: α=(∆t.P−Kn):[Kn (n 1 −1)]=0,51 logo, P H = ponde ao volume v=2×27,9−37,8 =18 cm3 do soluto
=−log [H+] = −log nα =1,3 e Ka=nα2/(1−α) =0,053. ácido, e é portanto y=0,038 g.
23
Vol. I, Fasc. 1 GAZETA DE FÍSICA Outubro, 1946

10 — 75 cm3 duma solução de permanganato de toras. A partir de 100 cm3 do soluto, correspondendo
potássio são empregados na oxidação de 3,15 g de a 10 g de matéria orgânica, obtiveram-se, em pre-
sulfato ferroso amoniacal cristalizado. sença dum excesso de hidroxilamina, 5 g dum deri-
Pede-se: a) quantos cm3 de soluto de Mn 04 K são vado cristalizado do metanal, continuando o resíduo
necessários para libertar todo o iodo de 1,95 g de aquoso a ser ácido e redutor. Pela fervura de outros
iodeto de potássio em solução ácida; b) que volume 100 cm3 do soluto original, destilou a 98,5° C um
de hipossulfito N/10 reduzirá êste iodo. R: a) Estabe- composto ácido e redutor, mostrando coloração ver-
lecidas as equivalências : melha com C13Fe, não precipitando com C12Ca e
MnO4K/5 <> (SO4)2Fe(NH4)2,6 OH2 <> IK deter- que foi neutralizado com 25 cm3 de OHNa, 0,55 N.
mina-se: v=109,6 cm3 do soluto de Mn O4K. b) A par- O resíduo da destilação, òpticamente inactivo, era
tir das equivalências: S 2 O 3 Na 2 <> I <> IK tem-se: azotado e foi titulado, após aplicação da técnica de
v′=117 cm3 de soluto de S 2 O 3 Na 2 , N/10. Sörensen, com 75 cm3 da mesma base. Qual a cons-
11— Adiciona-se ácido cloroplatínico em excesso a tituïção da mistura? R: Em 10 g da mistura, existem
uma solução de cloreto de amónio, calcinando-se for- 3,3 g de metanal, 0,63 g de metanoico e 3,1 g de
temente o precipitado obtido. Sendo 0,272 g o pêso glicocola.
do resíduo depois de aquecido, determine quanto
cloreto de amónio tinha a solução. Traduza por es- 14 — O extrato etéreo de 10 g dum produto vege-
quemas as reacções que se passam. tal, com 1,6 % de azoto, pesou 0,5 g. A extracção
R: p=0,148 g de Cl NH4. primeiro só com água e depois a quente com ácido e
alcali diluídos, deixou 1,0 g de resíduo sêco cuja
F.C.L.—Análise Química, 2.a parte—Julho de 1946 combustão deixou por sua vez 0,1 g de resíduo.
A extracção com água deu 50 cm3 de soluto aquoso
12 — 0,1 g dum diósido impuro, que é hidrolizável (extrato A); e a extracção com ácido e alcali diluí-
por um extrato de levedura e que reduz o licor de dos deu outros 50 cm3 de soluto (extrato B). A dé-
Fehling, é tratado com excesso de dinitrofenilhidra- cima parte de cada um dêstes solutos correspondeu a
zina, obtendo-se um precipitado de 0,132 g. a) Que 70 cm3 de licor de Fehling. No entanto, depois de
quantidade de hidrazina deve empregar; b) a que tratados com ClH diluído, 5 cm3 do extrato A cor-
conclusões qualitativas e quantitativas chega? (Esque- responderam a 80 cm3 de licor de Fehling, Que con-
mas da reacção). R a) 0,037 g de nitrofenilhidrazina. clusões tira sôbre a composição do produto? R: O
b) Trata-se de maltose com 35,5 % de impurezas. produto contém: 0,5 g de lipidos; 1,0 g de proteínas;
0,9 g de celulose; 3,5 g de glucose e ósidos-oses (expres-
13 — Um soluto aquoso, de procedência bioquímica, sos em gramas de glucose); 3,15 g de amido; 0,475 g
mostrou reacção ácida, produziu azoto em presença de sacarose ou ósidos-ósidos; 0,1 g de cinzas; e 0,375 g
de ácido azotos e revelou possuir propriedades redu- de outras substâncias, possivelmente água.

11. A FÍSICA NAS SUAS APLICAÇÕES


A FÍSICA E A ENGENHARIA CIVIL

MANUEL ROCHA
Foi o desenvolvimento da Física nos sécu- ramos, como, a Engenharia Civil, derivados
los XVII, XVIII e XIX que creou condições daquelas técnicas.
para o aparecimento, nêste último século, da Nota-se todavia um constante regresso no
Engenharia cuja característica essencial, em recurso à intuïção e ao empirismo à medida
face das antigas técnicas, é o uso que ela faz que vai progredindo o conhecimento das leis
de leis naturais quantitativas. naturais. Há já hoje muitos problemas de
As antigas técnicas baseavam-se na intuïção Engenharia relativamente aos quais o deter
e no empirismo, sendo contudo ainda hoje minismo das leis naturais à disposição do
fundamental o papel que êstes elementos engenheiro é, felizmente, tão apertado que
desempenham na Engenharia, sobretudo nos nenhuma margem lhe deixa para a iniciativa

24
Vol. I, Fasc. 1 GAZETA DE FÍSICA Outubro, 1946

pessoal. E digo felizmente porque, além de País onde se nota uma confiança desmedida
não ser de temer o marasmo que resultaria do no raciocínio: experimenta-se pouco e dis-
esgotamento dos problemas a resolver, o facto corre-se demais, o que tem conduzido a um
da solução ser imposta por leis tem a grande estreito cientismo livresco.
vantagem de simplificar a actuação do enge- Note-se que o contacto permanente do
nheiro e, o que é ainda mais importante, simul- Engenheiro com problemas humanos exige que
tâneamente permitir a máxima economia êle possua além da formação científica, para
dentro da segurança desejada. a qual a Física lhe dá a contribuição mais
A Engenharia Civil só se constituiu quando a importante, uma sólida formação humanístico-
Física lhe forneceu a Mecânica dos Meios -social. A importância da formação científica
Contínuos. e humanístico-social no ensino superior da
Deve frizar-se, contudo, que o engenheiro engenharia está bem patente nas sugestões
não recebe do físico, por via de regra, as leis que recentemente o «Committee on Engenee-
elaboradas de modo a poder aplicá-las direc- ring Education» da «American Society of Civil
tamente à resolução dos seus problemas. Engeneers» fez a 114 escolas americanas de
É necessária uma série de desenvolvimentos Engenharia Civil. Aconselha êsse «Commitee»
exigidos pela necessidade da sua extensão, que sejam dedicados ao ensino das ciências
mais ou menos legítima, a casos particulares (puras e aplicadas) e às disciplinas de huma-
que, não interessando ao físico, sejam impor- nidades e assuntos sociais, respectivamente
tantes nas aplicações. O engenheiro tem, 35% e 20% da duração total do curso; sobeja
digamos, que vencer bolsas de resistência portanto para o ensino das disciplinas da
deixadas pela frente de avanço do conheci- especialidade sòmente 45% daquela dura-
mento das leis naturais. Assim se consti- ção.
tuem as ciências técnicas ou aplicadas que, A actividade do engenheiro civil consiste
como a Resistência dos Materiais, a Hi- em dar aos materiais de construção à sua
dráulica, a Aerodinâmica Aplicada, a Ter- disposição, formas e dimensões tais que lhe
modinâmica Aplicada e a Electricidade Apli- permitam alcançar com segurança, economia
cada, são a pormenorização de ramos da e beleza o fim em vista: transpor um vale
Física que interessam especialmente à En- (ponte), reter as águas dum rio (barra-
genharia. gem), etc.
A importância da Física para o engenheiro Na escolha das formas a adoptar o enge-
não reside só no facto dela lhe fornecer meios, nheiro civil baseia-se essencialmente em cons-
as leis, para o conhecimento dos fenómenos, truções análogas já existentes e, além disso,
mas também na formação intelectual que o seu na sua intuïção ou sentido mecânico. É nesta
estudo confere. Efectivamente a Fisica aponta escolha que o engenheiro introduz na obra
ao engenheiro a atitude mental a tomar perante o seu poder creador.
um novo problema, o que é extremamente Escolhida a forma, entre as muitas que em
importante pois os problemas técnicos têm geral são possíveis, levanta-se o problema de
sempre facetas novas. prever se a construção se encontrará em boas
Para o ensino da Física poder atingir condições de segurança, isto é, se não se dará
estafinalidade êle deverá ser conduzido por a rotura quando a construção fôr submetida
via experimental, indutivo, e não por via às fôrças que sôbre ela actuarão, como o
axiomática. Esta via dá mesmo uma for- pêso próprio, as devidas à acção do ven-
mação contrária à que deve possuir o enge- to e de tremores de terra, as sobrecar-
nheiro pois êste deve estar sempre atento aos gas, etc.
resultados das experiências e à sua interpre- Aqui o engenheiro tem de recorrer à Física:
tação, O ensino experimental da Física, como à Mecânica dos Sólidos Deformáveis.
em geral o das ciências, é indispensável no nosso Desta constituiu a Física, em primeiro lugar,
25
Vol I, Fasc. 1 GAZETA DE FÍSICA Outubro, 1946

a Teoria da Elasticidade, depois dum labor Aquela noção de tensão é fundamental para
de dois séculos iniciado por Galileu em 1638 o engenheiro civil. Do facto, para averiguar
se uma construção se encontra em boas con-
dições de segurança êle precisa de conhecer,
em todos os pontos da construção, o estado
de tensão do qual se pode avaliar, pelo menos
numa primeira aproximação, o perigo de
rotura.
A Teoria da Elasticidade mostra que o
estado de tensão num ponto é definido por um
tensor simétrico de segunda ordem, e fornece
para a sua determinação um sistema de nove
equações às derivadas parciais de segunda
ordem. É na integração dêste sistema que se
encontra uma dificuldade, podemos dizer,
intransponível. De facto, são raros os casos
em que o sistema se pode integrar e, mesmo
assim, por vezes, com desenvolvimentos analí-
ticos muitíssimo longos o que desvaloriza a
solução pois, na Engenharia, só podem inte-
Gravura, representando uma viga encastrada num extremo e
ressar métodos de cálculo que exijam um
submetida no outro a uma carga (pêso), extraída da obra de Galileu trabalho cujo valor não exceda unia reduzida
Discorsi e Dimostrazioni Matematiche (Leiden, 1638), onde foram
apresentados os primeiros resultados com interesse para a estrutu-
fracção do custo da construção.
ração da Mecânica dos Sólidos Deformáveis. Em virtude daquela dificuldade houve neces-
sidade de criar um novo ramo de Mecânica
e rematado, pode bem afirmar-se, pelo estabe- dos Sólidos Deformáveis, a Resistência dos
lecimento das equações gerais, em 1821, por Materiais que melhor seria designar-se Resis-
Navier. tência das Construções. E à Física que se
A Teoria da Elasticidade baseia-se na deve o estabelecimento das suas leis funda-
noção de tensão e na lei de Hooke(1) que melhor mentais cujas conseqüências têm sido larga-
seria designar por hipótese pois, para a maior mente exploradas, desde há um século, pelos
parte das substâncias, as deformações não são engenheiros.
proporcionais às tensões. Essas leis são: a lei de Hooke, a lei de
Navier-Bernoulli que afirma manterem-se pla-
nas as secções transversais duma peça prismá-
(1) Hooke estabeleceu a sua famosa lei em 1660 e
tica quando submetida à flexão, e a lei da
p u b l i c o u - a e m 1 6 7 6 s o b a fo rm a d u m a n a g ra m a ,
distribuição linear das tensões tangenciais
«ceiiinosssttuu», do qual só em 1678 deu a solução:
« u t te n sio sie v is » . Pa ra f aze r uma idé ia do pa sso desenvolvidas na secção transversal dum cilin-
extraordinário dado por Hooke, bastará reparar numa dro submetido a uma torção. Para o enge-
comunicação, citada na página 4 da «History of the nheiro civil as mais importantes são as duas
Theory of Elasticity» de Todhunter e Pearson, apre-
primeiras, tendo sido o estabelecimento da
s e n ta d a e m 1 67 4 à Ro y a l S o c i e ty p o r S i r W i l l i a m
Petty. Nesta comunicação, que mostra a orientação
segunda que escorvou o enorme desenvolvi-
de alguns físicos (?) da época, o autor explica a elas- mento tomado pela Resistência dos Materiais.
ticidade dos sólidos por intermédio dum sistema com- A Física presta ainda um auxílio precioso
plicado de átomos aos quais atribui «propriedades para vencer a dificuldade, que apontámos, de
sexuais» baseando-se para isso numa passagem da
b íbl ia que d i z « ma cho e fê mea os c re ou» , a qu al o
integração de certas equações diferenciais da
autor afirma que deve aplicar-se às «partes últimas» Teoria da Elasticidade. De facto, recorrendo
da natureza e «portanto» aos homens e aos átomos. a outros fenómenos físicos que são regidos

26
Vol. I Fasc. 1 GAZETA DE FÍSICA Outubro, 1946

pelas mesmas equações diferenciais consegue- com freqüência sistemas com tão grande
se a integração experimental por analogia. número de equações que são inabordáveis
Assim, a integração experimental da equação pelos métodos analíticos conhecidos.
de Laplace, que interessa não só à Teoria da Os problemas de Resistência dos Materiais
que aparecem na Engenharia Civil são geral-
mente estáticos. Contudo também surgem
problemas dinâmicos, sobretudo relativos à
vibração de meios contínuos. Assim, há
necessidade de conhecer as tensões desenvol-
vidas numa ponte que vibra ao ser transposta
por um comboio, as desenvolvidas num edifí-
cio submetido à acção dum sismo, etc. Os
problemas dinâmicos só podem ainda, por via
de regra, ser tratados com métodos pouco
aproximados apesar do grande desenvolvi-
mento que a teoria das vibrações tem tido nos
últimos anos em virtude das necessidades da
construção de máquinas.
A Resistência dos Materiais por ser consti-
Montagem para a integração experimental da equação de Laplace tuída por um conjunto de teorias particulares,
a duas variáveis pois só são aplicáveis a sólidos com determi-
Na tina coloca-se uma placa, por exemplo de ebonite, na qual
,e fez uma abertura reproduzindo o domínio de integração, e nadas formas, não fornece muitas vezes os
enche-se a abertura com um líquido condutor. No contôrno desta valores das tensões e deslocamentos com a
abertura estabelecem-se potenciais proporcionais aos valores que
toma a função a integrar na fronteira do domínio de integração. aproximação necessária.
Os potenciais do líquido no interior da abertura são proporcio- Esta deficiência, apesar dos progressos da
nais aos valores da função.
Resistência dos Materiais, tem-se vindo acen-
Elasticidade mas também a outros domínios, tuando com o progresso da técnica e a
é sempre possível por intermédio da determi- conseqüente crescente preocupação de eco-
nação dos potenciais dum campo eléctrico nomia de materiais, que acerreta a necessidade
criado por condutores com formas e potenciais de conhecer as tensões desenvolvidas nas
escolhidos de harmonia com as condições aos construções com aproximação cada vez maior.
limites da equação de Laplace a integrar. Esta situação está obrigando, com crescente
Também é possível determinar as tensões freqüência, ao recurso a métodos experimen-
desenvolvidas numa estrutura constituída por tais para a determinação de tensões, quer
elementos rectilíneos, como a estrutura dum sôbre modelos das construções quer sôbre as
edifício, a partir da medição de certas gran- próprias construções.
dezas dum circuito eléctrico convenientemente É ainda à Física que o engenheiro recorre
escolhido, o que é conseqüência dos compor- — à semelhança mecânica — para a fixação
tamentos da estrutura e do circuito serem das dimensões a dar aos modelos, que por
regidos por certas condições de mínimo vezes não são geomètricamente semelhantes
formalmente idênticas. Dum modo geral é às construções reais, para a escolha dos mate-
possível resolver qualquer sistema de equa- riais dêsses modelos, que devem possuir
ções lineares a partir dum circuito eléctrico. propriedades relacionadas com as das constru-
Esta possibilidade foi recentemente aprovei- ções, e para o transporte para as construções
tada para a construção de máquinas eléctricas das tensões e deslocamentos observados sô-
destinadas à resolução de sistemas de equações bre os modelos quando lhe aplicamos fôrças
lineares, máquinas que terão certamente larga satisfazendo a condições fornecidas ainda pela
aplicação na Engenharia Civil onde aparecem semelhança mecânica.
27
Vol. I, Fasc. 1 G A Z E T A D E FÍ S IC A Outubro, 1946
A determinação experimental das tensões, o que tem interêsse nos casos de grandes
por meio de aparelhos designados extensó- espessuras, como sucede nas barragens que
metros, faz-se a partir da medição da defor- chegam a ter 200 m.
mação, isto é da variação da distância entre Nos modelos e, sobretudo nas construções,
dois pontos vizinhos, provocada pela aplicação interessa muitas vezes conhecer as tensões
das fôrças. provocadas pelas variações de temperatura.
Como as deformações a medir são muito Estas tensões sobrepõem-se às devidas às
pequenas, freqüentemente da ordem de gran- fôrças aplicadas e freqüentemente exce-
dem-nas. Para separar a contribuïção das
fôrças e das variações de temperatura para
as tensões observadas, torna-se indispensável,
posto que muitas vezes não seja suficiente,
conhecer a distribuïção de temperaturas. Para
isso, a Física põe à disposição do engenheiro
civil os termómetros de resistência eléctrica e
os pares termo-eléctricos que se aplicam na
superfície ou no interior das construções e
modelos.
Entre os métodos experimentais usados para
a determinação de tensões sôbre modelos tem
Modêlo da barragem de St. Luzia. Construído no Centro de
Estudos de Engenharia Civil (I. A. C.), para a determinação de
importância especial o baseado na Fotoelas-
tensões e deslocamentos provocados pela pressão hidrostática ticidade, capítulo da Física que nasceu com a
e por variações de temperatura.
A-modêlo do terreno descoberta de Brewster, em 1816, da birefrin-
B-modêlo da barragem
C-deflectómetros para medição dos deslocamentos
gência acidental de corpos transparentes
D-estrutura de suporte dos deflectómetros quando deformados.
E-cordas vibrantes
F-extensómetros mecânicos Neste método observam-se em luz polarizada,
O-pares termo-eléctricos
H-parte superior do espaço, a montante do modêlo, onde
sobe o mercúrio que exerce a pressão hidrostática.

deza do mícron, essa determinação experi-


mental apresenta sérias dificuldades. Até há
alguns anos eram quási exclusivamente usados
extensómetros constituídos por sistemas mecâ-
nicos que ampliam a deformação, mas, sobre-
tudo nas duas últimas décadas, tem-se recorrido
a extensómetros baseados na medição da
variação, provocada pela deformação, de
diversas grandezas físicas como, freqüência
própria de oscilação duma corda vibrante,
capacidade dum condensador, self duma bo-
bine, resistência dum circuito eléctrico e per- Figura de interferência obtida no estudo dum modêlo duma
meabilidade magnética. Com o quartzo piezo- viga apoiada sôbre duas colunas e que recebe unia carga por
intermédio doutra coluna. Estudo feito no Centro de Estudos
-eléctrico consegue-se a medição directa de de Engenhario Civil (I. A. C.).

tensões exercidas na superfície dum sólido.


Com alguns daqueles extensómetros conse- linear e circularmente, modelos transparentes,
gue-se mesmo fazer a determinação das tensões geralmente de vidro ou de plásticos, das cons-
no interior das construções ou dos modelos, truções. Quando se aplicam as fôrças ao

28
Vol I, Fasc. 1 GAZETA DE FÍSICA Outubro, 1946

modelo aparecem franjas de interferência, mais fundo, para estabelecer as leis gerais do
coloridas quando se trabalha em luz não fenómeno, as relações dêste com a estrutura
monocromática, das quais se pode deduzir da matéria.
completamente o estado de tensão no modelo, Com o estudo da plasticidade está ìntima-
no caso de se tratar dum equilíbrio a duas mente relacionado outro fenómeno, o da rotura
dimensões. Nos últimos 10 anos físicos e dos sólidos, cujo conhecimento é essencial
engenheiros têm-se esforçado, ainda sem para o engenheiro pois êle pretende sempre
completo sucesso, em generalizar o método evitar a rotura das construções.
da Fotoelasticidade para a determinação A Física do Estado Sólido tem-se ocupado
de tensões nos equilíbrios a três dimen- sobretudo do estudo da plasticidade e da
sões(1). O método da Fotoelasticidade tem rotura nos metais, sendo de grande interêsse
grande valor pedagógico em virtude de per- para o engenheiro os resultados obtidos nos
mitir um ensino visual da Mecânica dos Sólidos últimos vinte anos, pois êles vieram esclarecer
Deformáveis. muitos fenómenos que, apesar de conhecidos,
A Teoria da Elasticidade e a Resistência não podiam ser interpretados.
dos Materiais baseiam-se na hipótese de as A Física consegue calcular a tensão de
deformações serem perfeitamente elásticas, rotura e as constantes elásticas duma substân-
isto é reversíveis, e proporcionais às tensões cia desde que sejam conhecidas as fôrças
(lei de Hooke). Mas, por via de regra, mesmo actuantes entre os seus átomos ou iões. Mas
para as tensões relativamente baixas a que os constata-se que, enquanto é boa a concordân-
materiais estão correntemente submetidos nas cia entre as deformações calculadas e medidas,
construções, as deformações não são perfeita- os valores da tensão de rotura medidos são
mente elásticas, isto é, são em parte irrever- 1 1
sòmente cêrca de a dos valores
síveis, e não são proporcionais às tensões. 500 1000
Estas deformações, chamadas plásticas, são calculados(1). Se os valores das tensões de
por isso duma grande importância para a rotura dos sólidos fossem os dados pelo cál-
Engenharia Civil. culo, bastariam algumas cordas de piano para
Não se conhecem, contudo, as leis gerais suportar uma ponte suspensa e uma barragem
da deformação plástica apesar de, depois com uma centena de metros de altura bastaria
de Tresca em 1864 ter apresentado os pri- que tivesse alguns centímetros de espessura.
meiros resultados experimentais, muitos físicos Por aqui se poderá avaliar a extraordinária
que estudam o estado sólido se terem interes- repercussão que teria a produção industrial
sado por as estabelecer. de sólidos com tensões de rotura vizinhas das
Dado o grande interêsse que tem o assunto, previstas pela Física. E a esperança não é vã.
também numerosos engenheiros se têm dedi- De facto Joffé conseguiu obter com o sal
cado à investigação no campo da plasticidade. gema, ensaiado em certas condições, tensões
Porém, a orientação da investigação dos físi- de rotura de 160 Kg/mm2, muito superiores às
cos e engenheiros tem sido muito diferente, correntemente observadas e muito vizinhas da
como sucede sempre que atacam o mesmo teórica, que é de 200 Kg/mm2.
problema. De facto, o engenheiro o que deseja Ainda mais: com fios finos de vidro, obtidos
é estabelecer as relações de interêsse imediato, por estiragem conduzida sem quaiquer cuida-
em virtude das necessidades prementes das dos especiais, obtêm-se tensões de rotura mais
aplicações pois êle tem de construir quer elevadas do que com o melhor aço, com a
conheça ou não as leis. O físico pretende ir enorme vantagem para o construtor do peso

(1) Manuel Rocha, A fotoelasticidade nos equilí-

brios elásticos de revolução, Técnica, n.° 115 Dezem- Houwink, Elasticity Plasticity and Structure of
(1)

bro 1940. Matter, Camb. Univ. Press, 1937, Capítulo II.


29
Vol. I, Fasc. 1 G A Z E T A D E F Í S ICA Outubro, 1946

específico do vidro ser cerca de três vezes de pequenas fissuras em metais ferromagné-
menor do que o do aço. tico com auxílio dum campo magnético, e a
Dos trabalhos dos físicos sôbre o estado determinação das constantes elásticas com
sólido conclui-se que a causa do baixo valor auxílio de ultra-sons que, recentemente(1),
das tensões de rotura observadas reside em foram também utilizados para a determinação
defeitos da estrutura, designados por Smekal da posição das armaduras numa peça de betão
armado, determinacão que permite controlar
a perfeição com que a peça foi executada.
Não só a Mecânica dos Sólidos, à qual nos
temos vindo a referir, presta serviços na
resolução dos problemas de Engengaria Civil.
Também com freqüência há necessidade de
recorrer à Mecânica dos Líquidos, quer para
a determinação das acções dos líquidos sôbre
as construções, como, por exemplo, pressão
da água sôbre uma barragem, pressão das
ondas sôbre o molhe, quer para a determina-
ção das suas condições de escoamento em
cursos de água, canais, canalizações, ou ainda
para o estudo do transporte de materiais
sólidos em suspensão ou por arrastamento.
Na Mecânica dos Líquidos a Hidráulica está,
em relação à Hidrodinâmica, numa posição
análoga à da Resistência dos Materiais em
Fissuras superficiais reveladas por meio de vapor de sódio relação à Teoria da Elasticidade. Contudo,
no vidro Pirex, Resultado obtido em 1937 pelo físico inglês
Costa Andrade que assim confirmou experimentalmente a hipó-
tese, emitida muito antes, da existência de defeitos de estrutura.

por «Lockerstellen», pequenas fissuras que


determinam concentrações de tensões eleva-
díssimas. Portanto o problema que se põe é
êste: poder-se-ão evitar êsses defeitos?
Talvez não venha longe a época em que se
substitua o aço pelo vidro!
A Física do Estado Sólido tem ainda con-
corrido para dar coerência aos Ensaios de
Materiais, disciplina cuja finalidade é a deter-
Instalação de r aios X colocada par a a deter minação da po-
minação das propriedades dos materiais usados sição das armaduras numa construção de betão armado
nas construções, e que até há pouco era cons-
tituída exclusivamente por uma série de normas
enquanto na Mecânica dos Sólidos os proble-
empíricas. Além disso, vários ramos da Física
mas de maior interêsse para o engenheiro
têm concorrido para o progresso dos Ensaios
civil são estáticos, na Mecânica dos Líquidos
de Materiais. Assim, é hoje corrente a deter-
são dinâmicos. Desta circunstância resulta
minação de singularidades na estrutura dos
uma maior dificuldade na obtenção de solu-
materiais com raios X e gama (1) , a detecção

(1) Hahnshaw, The technique of gamma radiography, (1) Trabalho em curso no Eidgenössische Material-

Engeneering n.° 4206, 23 Agosto 1946, pág. 169. prüfunganstalt de Zürich.

30
Vol. I, Fasc. 1 GAZETA DE FÍSICA Outubro, 1946

ções satisfatórias, sendo por isso muito fre- fornecidos pela Física acerca dos fenómenos
qüente a necessidade de recorrer na Hidráulica de superfície nos líquidos e sólidos, particular-
a resultados empíricos. mente dos fenómenos capilares.
Há um outro capítulo de Mecânica, de cons- Vem a propósito referir que as tensões
tituição muito recente, com grande interêsse provenientes dos fenómenos capilares que se
para a Engenharia Civil. É a chamada Meca- produzem nas construções de betão e nas de
nica dos Solos que estuda os sistemas de madeira, materiais que são atravessados por
pequenas partículas, como argilas e areias. uma rede de canais capilares, são muitas vezes
O seu interêsse resulta do facto das funda- responsáveis pelas fracturas (fendas) que fre-
ções das construções serem com freqüência qüentemente se observam, pois essas tensões
feitas sôbre terrenos com tal contituïção e, podem ser muito mais elevadas do que as
além disso, de êles servirem como material devidas às fôrças aplicadas.
para a construção de certas obras como as Antes de acabar esta breve exposição acerca
chamadas barragens de terra. das relações entre a Física e a Engenheria
Coulomb, Navier e Rankine estabeleceram Civil, não posso deixar de citar o concurso
os primeiros resultados da Mecânica dos Solos, daquela ciência para a melhoria das condições
tratando-os à luz dos resultados clássicos da de habitabilidade dos edifícios, sem o qual não
Mecânica dos Sólidos Deformáveis. Foi só teria sido possível o condicionamento automá-
nas duas últimas décadas, depois de um tico do ar ambiente quanto a temperatura e
engenheiro, Terzaghi, ter feito voltar as aten- humidade, a boa iluminação e as boas condi-
ções para o papel importante que desempe- ções acústicas. Para fugir às irregularidades
nham os filmes líquidos que separam as das condições atmosféricas, nota-se mesmo
partículas dos solos, que a Mecânica dos hoje a tendência para o edifício sem janelas
Solos entrou em franco desenvolvimento, só no interior do qual as condições podem ser
possível, aliás, graças aos conhecimentos reguladas à vontade.
MANUEL ROCHA
ENGENHEIRO CIVIL

12. INFORMAÇÕES VÁRIAS


EFEMÉRIDES sores do ensino secundário e do ensino superior. As
1745—Nasceu Alessandro Volta (falecido em 1827). vantagens resultantes de uma tal aproximação são
Em 1800 descrição da 1.ª pilha eléctrica. tantas que em alguns países, como por exemplo na
1845—Nasceu, em 27 de Março, Wilhelm Conrad Suécia e no México, os professores do ensino secun-
Röntgen (falecido em 10 de Fevereiro de 1923. dário são pagos pelo Estado para irem, de cinco em
Em 1895 (Novembro, 8) descoberta dos raios X. cinco anos, passar um ano à Universidade. E não se
1845—Nasceu Gabriel Lippmann (falecido em 31 de pense que dêste contacto resultará só vantagem para um
Julho de 1921). dos lados; se é certo que o professor do ensino secun-
Em 1895 primeira fotografia a côres com dário, vindo à Universidade comparticipar nos semi-
perfeito ortocromatismo. nários e nos colóquios e realizar trabalhos práticos
1646—Nasceu Gottfried-Wilhelm Leibnitz (falecido ou mesmo investigação, vê assim facilitada a sua
em 1716). tarefa de uma permanente actualização de conheci-
mentos, não menos certo é que, com a experiência
NOTICIÁRIO adquirida no ensino aos mais jóvens, poderá
e deverá exercer junto do professor universitário uma acção
Cursos práticos para professores permanente no sentido de que êste tenda a modificar
Um dos factos que mais mais pode contribuir para os seus programas com o objectivo de preencher
o desenvolvimento do meio científico de um país é o aquelas lacunas que o professor do ensino secundário
da existência de um estreito contacto entre os profes- tenha reconhecido possuir na sua preparação univer-

31
Vol. I, Fasc. 1 GAZETA DE FÍSICA Outubro, 1946

sitária. É mesmo corrente que dêste intercâmbio experimental dêsse trabalho realizou-a o autor, como
resulte a passagem para o corpo docente universitário bolseiro do Instituto para a Alta Cultura, no Centro
de alguns professores do ensino secundário cujas de Estudos de Física que funciona no Laboratório
aptidões melhor poderão ser aproveitadas no grau de de Física da Faculdade de Ciências de Lisboa.
ensino superior. Também recentemente se doutorou em Zürich o assis-
Ora acontece que entre nós tem havido uma disso- tente da Faculdade de Ciências de Lisboa, Armando
ciação pode dizer-se completa entre os professores Carlos Gibert. O Dr. Armando Gibert apresentou
dos dois graus de ensino; e essa separação constitui, sem como tese de doutoramento um trabalho intitulado
dúvida, um grave prejuizo para a Nação. Com o fim «Effet de la température sur la diffusion neutron-
de, embora com um carácter restrito, contribuir para proton» que realizou no Instituto de Física, da direc-
realizar uma aproximação que tão útil seria, o Labo- ção do Professor P. Scherrer, da Escola Politécnica
ratório de Física da Faculdade de Ciências de Lisboa de Zürich, onde permaneceu durante quatro anos como
organiza, no corrente ano lectivo, um curso prático bolseiro do Instituto para a Alta Cultura. M. V.
em que os professores do ensino secundário terão
ocasião de executar alguns trabalhos de física que «Portugaliae Physica»
muitos deles não realizaram quando freqüentaram as
Faculdades. Será posto à venda, antes do fim do ano, o 2.° fas-
A Gazeta de Física terá ocasião, no seu próximo cículo do 2.° volume da revista Portugaliae Physica;
número, de se referir pormenorizadamente a esta êste fascículo insere colaboração nacional e estrangeira.
iniciativa. M. V. Chamamos a atenção dos nossos leitores para o
facto da comissão de redacção da referida revista ter
Curso de Ciências Geofísicas resolvido baixar o preço de assinatura, que foi de
O decreto-lei n.° 35850, publicado no «Diário do Esc. 150$00 para o 1.° volume, para Esc. 80$00.
Govêrno», 1.ª Série, de 6 de Setembro p. p. modifica Como o 2.° volume deverá ser publicado no biénio
o quadro das disciplinas das Faculdades de Ciências 1946-47 um encargo anual de quarenta escudos per-
e, em particular: mitirá a todos os estudiosos portugueses contribuirem
1.°) Introduz as seguintes modificações na actual para a manutenção de uma revista que publica o seu
licenciatura em Ciências Físico-Químicas: segundo volume sem qualquer auxílio financeiro do
a) As cadeiras de Física dos Sólidos e Fluídos Estado.
(anual) e de Acústica, Óptica e Calor (anual) são Dados os serviços que esta revista presta à cultura
extintas e criam-se as cadeiras de Mecânica Física nacional — em especial ao movimento incipiente de
(semestral) e de Óptica (anual). investigação científica no domínio da física — e
b) O Curso Geral de Física (anual) passa a ser ainda à projecção dessa mesma cultura no estrangeiro,
precedência obrigatória das cadeiras de Mecânica seria de lamentar que dificuldades de ordem financeira
Física, Termodinâmica, Electricidade e Óptica. obrigassem à suspensão da sua publicação. M. V.
c) Extingue-se a cadeira Geografia Física e
Física do Globo (anual) que será substituiria pelo Novas unidades
curso de Geomorfologia (semestral). Os físicos E. U. Condon e L. F. Curtiss acabam de
2.°) Institui o «curso de Ciências Geofísicas» com propor as seguintes novas unidades:
8 semestres constituido pelas seguintes disciplinas: 1.° O «rutherford», abreviação «rd», que é a inten-
1:º ano : Matemáticas Gerais (ou Algebra Superior) sidade duma fonte radioactiva na qual se dão 106
C. G. de Física, C. G. de Mineralogia e Geologia, desintegrações por segundo.
Desenho de Máquinas. O micro-rutherford é pois uma desintegração por
2.° ano: Cálculo infinitesimal, Mecânica Física, segundo. É uma unidade distinta, por natureza, do
Termodinâmica, C. G. de Química, Desenho Topo- curie.
gráfico. 2.º) O «roentgen por hora a 1 metro», abreviação
3.° ano : Análise Superior, Cálculo das Probabili- «r. h. m.», pronúncia «rum». É uma unidade apro-
dades, Mecânica Racional, Electricidade. priada à medição de intensidades de radiação gama.
4.° ano : Física Matemática, Óptica, Metereologia O «National Bureau of Standards» dos Estados
e Geofísica. A. G. Unidos da América do Norte recomenda a adopção
destas unidades. A. G.
Doutoramentos
Agradecimento
Doutorou-se, no mês de Junho, em ciências físico-
-químicas na Faculdade de Ciências do Pôrto o assis- Ao terminar a composição do 1.° número da Gazeta
tente de física desta Faculdade, José Sarmento de de Física, em 22 de Outubro de 1946, a Direcção
Vasconcelos e Castro. O Dr. José Sarmento apresen- agradece a todos aquêles que confiaram no êxito da
tou como tese de doutoramento um trabalho intitulado sua realização, como se prova pela inscrição de quási
«Estudo das riscas satélites de Lα do ouro»; a parte duzentos assinantes até esta data. X. B.
32
PORTUGALIAE PORTUGALIAE
MATHEMATICA
ACTA BIOLÓGICA
REVISTA DE
COLABORAÇÃO INTERNACIONAL
{
Preço dos volumes já publicados

Volume 1 — 300$00 Publica trabalhos originais


Volumes 2, 3 e 4 — 250$00 cada
ou revisões críticas de
Para os sócios da Sociedade problemas actuais de Biologia
Portuguesa de Matemática:

Volume 1 — 200$00 {
cada um dos volumes seguintes: 150$00

Editada como publicação do Instituto


z
Botânico da Faculdade de Ciências
Assinatura do volume 5: 150$00 de Lisboa, do Laboratório de Patologia
e para os sócios da S. P. M. 50$00 Vegetal «Verissimo de Almeida»,
Pedidos a: Gazeta de Matematica, Lda. da Sociedade Portuguesa de Biologia
Ä Ä

PORTUGALIAE PHYSICA
Revista de colaboração internacional

REDACÇÃO E ADMINISTRAÇÃO
Laboratorio de Física da Faculdade de Ciências de Lisboa

Publicados: Volume 1 (4 fascículos)


e Volume 2 (fascículo 1)
No prelo: Volume 2 (fascículo 2)
Assinatura do Volume 2: Esc. 80$00

Para os sócios da Sociedade Portuguesa de Física e Química, redução de 50%


Ä

Ä Os anúncios com êste sinal não são pagos.

Dêem o vosso apoio à investigação científica


INTRUMENTOS D I V E R S O S
O F T Á L M I C O S Binóculos .
Lupas
Oftalmoscópios binoculares
Lentes para óculos
K e r a t ó m e t r o s
Óculos para o Sol Lentes fotográficas

MICROSCÓPIOS EQUIPAMENTOS
E ACESSÓRIOS DE PROJECÇÃO
Hemacitómetros -Micrótomos Projectores de, contorno
Máquinas para microfo- Microprojectores—Aparelhos
tografia e acessórios de projecção (Epidiascópios)

INSTRUMENTOS PARA AEROFOTOGRAMETRIA


Lentes Metrogon Aparelhagem Multiplex

INSTRUMENTOS PARA MEDIR


PROPRIEDADES ÓPTICAS
Refractómetros — Polarímetros Sacarímetros — Equipamentos
espectrográficos — Espectrofotómetros

BAUSCH AND LOMB OPTICAL Cº.


REPRESENTADA EM PORTUGAL PELO

INSTITUTO PASTEUR DE LISBOA

Anunciar na «Gazeta de Física» é contribuir para a sua prosperidade

Você também pode gostar